NSG 100 Exam 1 Flashcards

1
Q

A student nurse is studying clinical judgment theories and is working with Tanner’s Model of Clinical Judgment. How can the student nurse best generalize this model?

a. A reflective process where the nurse notices, interprets, responds and reflects in action
b. One conceptual mechanism for critiquing ideas and establishing goal-oriented care
c. Researching best practice literature to create care pathways for certain populations
d. Assessing, diagnosing, implementing, and evaluating the nursing care plans

A

a. A reflective process where the nurse notices, interprets, responds, and reflects in action

Rationale: Looking across theories and definitions of clinical judgment, they all have in common the ability to reflect on data and choose actions.
Reflection also considers evaluating the result of the actions to determine whether they were effective. Although critiquing ideas and establishing goal-oriented care could be considered part of a generalized statement of critical thinking, this is not broad enough without reflection and evaluation. Clinical judgment would most likely be used to create care paths derived from the evidence; however, this is not the cornerstone of the Tanner Model. Clinical judgment is used when engaging in the nursing process, but this is too narrow in focus to capture the essence of critical thinking definitions and theories. Critical thinking is not synonymous with the nursing process.

How well did you know this?
1
Not at all
2
3
4
5
Perfectly
2
Q

The nurse is implementing a plan of care for a patient newly diagnosed with type 2 diabetes mellitus. The plan includes educating the patient about diet choices. The patient states that they enjoy exercising and understand the need to diet; however, they can’t
see living without chocolate on a daily basis. Using the principles of responding in the Model of Clinical Judgment, how would the nurse proceed with the teaching?

a. The nurse explains to the patient that chocolate has a high glycemic index. The nurse then focuses on foods that have low glycemic indexes and provides a list for the patient to choose from.
b. The nurse explains that the patient may eat whatever they would like as long as the patient’s glucose reading and A1c remain stable.
c. The nurse derives a new nursing diagnosis of Knowledge Deficit and readjusts the plan of care to include additional sessions with the registered dietician.
d. The nurse examines the patient’s daily glucose log and incorporates the snack into the time of day that has the lowest readings. The nurse then follows up and evaluates the response in 1 week.

A

d. The nurse examines the patient’s daily glucose log and incorporates the snack into the time of day that has the lowest readings. The nurse then follows up and evaluates the response in 1 week.

Rationale: Responding entails adjusting the plan of care to the particular patient issue through one or more nursing interventions. In this case, the nurse is working with the patient’s wishes, knowing that the patient will most likely cheat. The patient will be allowed to “cheat.”
The plan will be evaluated to be sure the snack does not elevate the glucose excessively and be readjusted if warranted.
While it is true that most chocolate has a high glycemic index, providing a list of foods that do not include the one thing the patient enjoys will most likely lead to nonadherence to the diet. Advising the patient that they can have whatever they want to eat may lead to further dietary indiscretions and cause side effects such as obesity or high glucose readings.
Knowledge Deficit is an inaccurate diagnosis for this patient as evidenced by the patient stating they understand the need to exercise and the need to diet.

How well did you know this?
1
Not at all
2
3
4
5
Perfectly
3
Q

A new graduate nurse is working with an experienced nurse to chart assessment findings. The new nurse notes that the physical therapist wrote on the chart that the patient is lazy and did not want to participate in assigned therapies this AM. The experienced
nurse asks the new nurse what may be going on here. What is the best explanation for this statement?

a. Data on the chart can sometimes be documented in a biased manner.
b. Data on the chart changes as the patient’s condition changes.
c. Data on the chart is usually accurate and can be verified by the patient.
d. Reading the chart is not a wise use of time as this can be time-consuming and tedious.

A

a. Data on the chart can sometimes be documented in a biased manner.

Rationale: It is important that the nurse records only what is assessed, without adding judgments or interpretations to the record.
Data do indeed change (and need to be charted) as the patient’s condition changes, but this would not be the best answer to this question.
Assessment data may at times be difficult to obtain, but that would not occur often enough to warrant a warning about the difficulty in charting it. Also, obtaining data is clearly a different activity from charting it.
Charting can be time-consuming and tedious, but this is not the most complete answer to this question.

How well did you know this?
1
Not at all
2
3
4
5
Perfectly
4
Q

A home care nurse receives a physician order for a medication that the patient does not want to take because the patient has a history of side effects from this medication. The nurse carefully listens to the patient, considers it in light of the patient’s condition, questions its appropriateness, and examines alternative treatments. What is the nurse’s best action?

a. Call the physician, explain the rationale, and suggest a different medication.
b. Consult an experienced nurse on whether there are other similar treatments.
c. Hold the drug until the physician returns to the unit and can be questioned.
d. Question other staff as to the physician’s acceptance of nursing input.

A

a. Call the physician, explain the rationale, and suggest a different medication.

Rationale: Determining how best to proceed on behalf of a patient’s best health outcomes care may require clinical judgment. At the committed level of critical thinking, the nurse chooses an action after all possibilities have been examined.
A home care nurse who is using good clinical judgment techniques should have confidence in their decision and may not have another nurse available as this is an autonomous setting.
Holding the drug might jeopardize the patient’s health, so this is not the best solution.
The nurse working at this level of critical thinking makes choices based on careful examination of situations and alternatives; whether or not the physician is open to nursing input is not relevant.

How well did you know this?
1
Not at all
2
3
4
5
Perfectly
5
Q

A patient has been admitted for a skin graft following third-degree burns to the bilateral calves. The plan of care involves 3 days inpatient and 6 months outpatient treatment, including home care and dressing changes. When should the nurse initiate the educational plan?

a. After the operation and the patient is awake
b. On admission, along with the initial assessment
c. The day before the patient is to be discharged
d. When narcotics are no longer needed routinely

A

b. On admission, along with the initial assessment

Rationale: Initial discharge planning begins upon admission.
After the operation has been completed is too late to begin the discharge planning process.
The day before discharge is too late for the nurse to gather all pertinent information and begin teaching and
coordinating resources.
After a complicated operation, the patient may well be discharged on narcotic analgesics. Waiting for the patient to not need them anymore might mean the patient gets discharged without teaching being done.

How well did you know this?
1
Not at all
2
3
4
5
Perfectly
6
Q

A nurse has designed an individualized nursing care plan for a patient, but the patient is not meeting goals. Further assessment reveals that the patient is not following through on many items. Which action by the nurse would be best for determining the cause of the problem?

a. Assess whether the actions were too hard for the patient.
b. Determine whether the patient agrees with the care plan.
c. Question the patient’s reasons for not following through.
d. Reevaluate data to ensure the diagnoses are sound.

A

b. Determine whether the patient agrees with the care plan.

Rationale: Having the patient and/or family provide input to the care plan is vital in order to gain support for the plan of action. The actions may have been too difficult for the patient, but this is a very narrow item to focus on.
The nurse might want to find out the rationale
for the patient not following through, but instead of directly questioning the patient, which can sound accusatory, it would be best to offer some possible motives.
Reevaluation should be an ongoing process, but the more likely cause of the patient’s failure to
follow-through is that the patient did not participate in making the plan of care.

How well did you know this?
1
Not at all
2
3
4
5
Perfectly
7
Q

A new nurse appears to be second-guessing herself and is constantly calling on the other nurses to double-check their plan of care or rehearse what they will say to the doctor before she calls on the patient’s behalf. This seems to be annoying some of the nurse’s coworkers. What is the nurse manager’s best response?

a. Explain to coworkers that this is a characteristic of critical thinking and is important for the new nurse to improve reasoning skills.
b. Agree with the staff and have someone follow and work more closely with a preceptor.
c. Have a talk with the nurse and suggest asking fewer questions.
d. Tell the staff that all new nurses go through this phase, and ignore their behavior.

A

a. Explain to coworkers that this is a characteristic of critical thinking and is important for the new nurse to improve reasoning skills.

Rationale: Reflection-on-action is critical for the development of knowledge and improvement in reasoning. It is where learning from practice is
incorporated into the experience. Inquisitiveness is a characteristic of critical thinking and reflects a desire to learn even when the knowledge may not appear readily useful. The manager should promote this. Suggesting the nurse work more closely with a
preceptor implies that the manager thinks the nurse needs to learn more and increase confidence. In reality, this nurse is demonstrating a characteristic of critical thinking.
Suggesting that the nurse ask fewer questions would hamper the development of the nurse as a critical thinker.
All new nurses do go through a phase of asking more questions at one time, but dismissing the nurse’s behavior with this explanation is simplistic and will discourage critical thinking

How well did you know this?
1
Not at all
2
3
4
5
Perfectly
8
Q

A nurse has committed a serious medication error and has reported the error to the hospital’s adverse medication error hotline as well as to the unit manager. The manager is a firm believer in developing critical thinking skills. From this standpoint, what action by the manager would best nurture this ability in the nurse who made the error?

a. Have the nurse present an in-service related to the cause of the error.
b. Instruct the nurse to write a paper on how to avoid this type of error.
c. Let the nurse work with more experienced nurses when giving medications.
d. Send the nurse to refresher courses on medication administration.

A

a. Have the nurse present an in-service related to the cause of the error.

Rationale: Nurturing critical thinking skills is done in part by turning errors into learning opportunities. If the nurse presents an in-service on the cause and prevention of the type of error committed, not only will the nurse learn something but many others nurses on the unit will learn from it too. This is the best example of developing critical thinking skills.
This option would allow the nurse to learn from the mistake, which is a method of developing critical thinking skills, but the paper would benefit only the nurse, so this option is not the best choice.
Letting the nurse work with more experienced nurses might be a good option in a very limited setting, for example, if the nurse is relatively new and the manager discovers a deficiency in the nurse’s orientation or training on giving medications in that system. Otherwise, this option would not really be beneficial. Sending the nurse to refresher courses might be a solution, but it is directed at the nurse’s learning, not critical thinking. The nurse might feel resentful at having to attend such classes, but even if they were helpful, only this one nurse is learning. Going to generic classes also does not address the specific reason this error occurred, and thus might be irrelevant. Critical thinking and learning can be enhanced by a presentation to the staff on the causes of the error.

How well did you know this?
1
Not at all
2
3
4
5
Perfectly
9
Q

A nurse is caring for a patient in a long-term care facility who has not been sleeping well. She notes that the patient is new to the facility, has been refusing therapy, and is also not eating well. The nurse interprets this to mean that the patient has been having trouble adjusting. The nurse decides to meet with the patient’s care team. The team decides to assess the patient’s willingness to participate in group recreational activities. The patient agrees to participate. After 1 week, the nurse reevaluates the plan of care and notes that the patient has been sleeping much better. Which of the following terms best describe processes used in the nurse’s plan? (Select all that apply.)

a. Clinical judgment
b. Evidence-based practice
c. The nursing process
d. Collaborative care planning
e. Positive reward process

A

a. Clinical judgment
c. The nursing process
d. Collaborative care planning

Rationale: Clinical judgment is a reflective process by which the nurse notices, interprets, responds, and reflects in action.
The nursing process is a process by which the nurse assesses, diagnoses, implements, and evaluates the nursing care plan.
Consulting and gaining input from the healthcare team is collaborative care planning.
Evidence-based practice refers to using interventions found in research studies.
The positive reward process is not a term used in care planning.

How well did you know this?
1
Not at all
2
3
4
5
Perfectly
10
Q

The nurse is reviewing the last 3 days of a patient’s pain history and notes that the pain level has remained constant. The nurse validates the pain level with the patient and decides to contact the provider for further orders. In this scenario, which process is the nurse is using?

a. Decision-making
b. Reasoning
c. Problem-solving
d. Judgment

A

d. Judgment

Rationale: Processes dependent on critical thinking include problem-solving, decision making, reasoning, and judgment. Judgment is the process of forming an opinion by comparing solutions through reasoning. The nurse observes that the patient’s pain level is not
decreasing and further assesses the pain level through discussions with the patient. The nurse concludes that the patient’s pain should be further addressed and contacts the provider. Decision-making requires choosing a solution to a problem. The student is
making a decision by reviewing two pertinent resources related to the removal of staples. Reasoning is the process by which a nurse links thoughts, ideas and facts together in a logical way. A systematic approach in finding solutions is termed problem
solving.

How well did you know this?
1
Not at all
2
3
4
5
Perfectly
11
Q

The nurse has been hired for a first job and is nervous about making errors in clinical judgment. It is important for the nurse to realize that clinical reasoning and the ability to make decisions in a clinical setting occurs at which time?

a. When it has been instilled in the content covered in nursing school.
b. When it is solely based in clinical experience.
c. When it develops over time with increased knowledge and expertise.
d. When it is an expectation of all nurses regardless of experience.

A

c. When it develops over time with increased knowledge and expertise.

Rationale: Clinical reasoning uses critical thinking, knowledge, and experience to develop solutions to problems and make decisions in a clinical setting. A nurse’s clinical-reasoning skills develop over time with increased knowledge and expertise.

How well did you know this?
1
Not at all
2
3
4
5
Perfectly
12
Q

The nurse is taking an advanced cardiac life support (ACLS) recertification class. As part of that class, the nurse and other nurses in the group rotate responsibilities during multiple mock code exercises simulating cardiac arrest scenarios. The nurse recognizes what process is assigning the nurses to these different responsibilities?

a. Concept mapping
b. Simulation
c. Role playing
d. Literature review

A

c. Role playing

Rationale: A role-play strategy involves assigning learners to different roles based on expected outcomes in a particular setting. Other learners
and facilitators observe the role-playing, and then all are involved in the debriefing or discussion of the scenario. As with simulation, this approach allows learners to interact in a safe, controlled environment. The concept map is a way to organize and visualize data to identify relationships and solve problems. Simulated experiences enable the student to apply previously learned content in a safe and realistic environment that allows time for questioning, clarifying, and feedback. Students develop confidence
in providing direct nursing care. Because critical thinking cannot occur about subjects that are unknown, a review of literature may foster this type of thinking by addressing knowledge deficits.

How well did you know this?
1
Not at all
2
3
4
5
Perfectly
13
Q

The nurse is preparing to administer an anticoagulant when the patient says, “Why do I have these bruises on my arms?” The nurse reviews the patient’s blood tests and notes an abnormal bleeding time. When the nurse then decides to hold the medication and notify the health care provider, the nurse recognizes this to be an example of which action?

a. Thinking aloud
b. Reviewing the literature
c. Applying knowledge
d. Role playing

A

c. Applying knowledge

Rationale: Nursing practice is based on the application of knowledge to address patient problems. In this case, the nurse is able to connect the medication, physical signs and laboratory data to determine a course of action. Nurses may “think aloud” as an inner dialogue to examine their thinking. The literature review is used to address knowledge gaps through the review of scholarly journals. A role-play strategy involves assigning learners to different roles based on expected outcomes in a particular setting. Other learners and facilitators observe the role-playing, and then all are involved in the debriefing or discussion of the scenario.

How well did you know this?
1
Not at all
2
3
4
5
Perfectly
14
Q

The nurse is preparing to restart a patient’s intravenous line and discovers that the patient has no usable veins in either arm. When working to solve this problem, the nurse should carry out which action?

a. Discuss the problem with the nurse in charge.
b. Not start the intravenous line.
c. Conduct an Internet search for infusion journal articles.
d. Contact the provider and report the concern.

A

a. Discuss the problem with the nurse in charge.

Rationale: Whether in an academic setting or in the clinical area, discussion of a problem, issue, or situation with colleagues may improve critical thinking. Through dialogue with others who have expertise or experience with the issue being faced, knowledge gaps can be filled, erroneous assumptions exposed, and unconscious biases addressed. Not starting the intravenous line is not an option at this point. A literature review to gain published information about intravenous complications may be appropriate after the patient’s concern has been addressed. Initially contacting the provider without fully exploring the options for alternate insertion sites is neither wise nor recommended.

How well did you know this?
1
Not at all
2
3
4
5
Perfectly
15
Q

The nurse has finished a shift and is on the way home. During the shift, one of the patients attempted to climb out of bed and fell. When the nurse is returning home and is thinking about what could have been done differently to prevent the fall, this would be an example of what concept?

a. Evidence
b. Standards
c. Attributes or traits
d. Reflection

A

d. Reflection

Rationale: A reflection is an effective tool that enables students and nurses to think about how best to improve their future caregiving in similar situations. The results of deliberate thinking are used to guide further thinking. Identification and use of evidence is necessary to guide analysis of situations and decision making. Nursing practice must be based on evidence gained through research and review of findings. Some personal characteristics are associated with critical thinking. Critical thinking needs to be assessed and evaluated according to standards to ensure the quality of thinking. Nursing practice is based on standards established by the American Nurses Association in areas such as the nursing process, ethics, education, research, communication, leadership, and collaboration.

How well did you know this?
1
Not at all
2
3
4
5
Perfectly
16
Q

When working on the ability to critically think, the nurse needs to develop a critical-thinking character that includes which quality?

a. Developing honesty and confidence
b. Learning from experiences
c. Enhancing self-reliance
d. Growing a “thick skin” to withstand criticism

A

a. Developing honesty and confidence

Rationale: To develop critical thinking, the nurse needs to develop a critical-thinking character, which includes maintaining high standards and developing critical-thinking qualities such as honesty, fair-mindedness, creativity, patience, persistence, and confidence. The
next step in the development of critical thinking includes taking responsibility for personal learning and seeking needed experiences that can provide the necessary knowledge on which to base the thinking. Fostering interpersonal skills, such as teamwork, conflict management, and advocacy, is important in the development of critical thinking. Self-evaluation and having thinking evaluated by others require the ability to accept and use constructive criticism.

How well did you know this?
1
Not at all
2
3
4
5
Perfectly
17
Q

A patient arrives at the urgent care clinic and complains of vague pains in the legs and the nurse asks the patient to describe this pain in greater depth. The nurse knows this is a critical-thinking skill and can be developed in which context?

a. Critical thinking is used to avoid repetition in providing care.
b. Critical thinking can be enhanced through practice.
c. Critical thinking should be based in thought and not spontaneity.
d. Critical-thinking skills become dull when used routinely.

A

b. Critical thinking can be enhanced through practice.

Rationale: The ultimate goal is for these questions to become so spontaneous in thinking that they form a natural part of our inner voice, guiding us to better reasoning. As with any skill, critical thinking can be enhanced through practice. The routine use of these
questions should promote critical thought.

How well did you know this?
1
Not at all
2
3
4
5
Perfectly
18
Q

The nurse is planning care for a group of patients and recognizes which activity may be delegated to unlicensed assistive personnel?

a. Analysis of the patient’s physical condition
b. Morning vital signs, height, and weight
c. Evaluation of whether colostomy drainage is normal
d. Determining patient readiness for post-surgical learning

A

b. Morning vital signs, height, and weight

Rationale: The nurse often works with unlicensed assistive personnel (UAP) to collect relevant data on height and weight, intake and output, and vital signs. The registered nurse uses critical thinking to guide decisions related to delegation of assignments and tasks. Before delegation of a task, the nurse must be knowledgeable about the role, scope of practice, and competency of the recipient of the delegated task. Analysis and evaluation of patient conditions and readiness for teaching require critical thinking and are nursing functions.

How well did you know this?
1
Not at all
2
3
4
5
Perfectly
19
Q

The nurse is preparing to teach indwelling urinary catheter insertion techniques to a group of graduate nurses. Which teaching-learning strategy would the nurse find most useful in teaching this skill?

a. Concept mapping
b. Simulation
c. Role playing
d. Literature review

A

b. Simulation

Rationale: Simulated experiences enable the student to apply previously learned content in a safe and realistic environment that allows time for questioning, clarifying, and feedback. Students develop confidence in providing direct nursing care. The concept map as a way to organize and visualize data to identify relationships and solve problems.
Role-play strategies involve assigning learners to different roles based on expected outcomes in a particular setting. Other learners and facilitators observe the role-playing, and then all are involved in the debriefing or discussion of the scenario.
As with simulation, this approach allows learners to interact in a safe, controlled environment. Because critical thinking cannot occur about subjects that are unknown, a review of literature may foster this type of thinking by addressing knowledge deficits.

How well did you know this?
1
Not at all
2
3
4
5
Perfectly
20
Q

The nurse is administering medications to a patient with high blood pressure. The patient states, “This pill made me so sick yesterday. Are you sure I have to take it now?” What action does the nurse take next?

a. Give the medication because no one gets sick on this pill.
b. Hold the medication and check the order since there may be a lack of information.
c. Give the medication since he/she is the nurse and knows what should be done.
d. Give the medication since the nurse did not see the provider come so the order is valid.

A

b. Hold the medication and check the order since there may be a lack of information.

Rationale: Errors in thinking can lead to errors in nursing care. It is essential for the nurse to assess and validate the patient’s concerns before proceeding with planned action. This helps avoid decisions being made on personal biases caused by preconceived notions. The nurse should hold the medication and investigate further. Believing that “no one gets sick on this pill” is a preconceived bias.
Giving the medication because “I know best” is an example of close-mindedness. Assuming the order is valid simply because the nurse did not see the provider is illogical thinking.

How well did you know this?
1
Not at all
2
3
4
5
Perfectly
21
Q

The nurse recognizes that when a patient is initially interviewed and assessed, the nurse must complete which tasks? (Select all that apply.)

a. Analyze the patient’s psychomotor status.
b. Take the patient’s vital signs.
c. Weigh the patient using a bed scale.
d. Evaluate the patient’s emotional and spiritual needs.
e. Ensure the coordination of the patient’s care.

A

a. Analyze the patient’s psychomotor status.
d. Evaluate the patient’s emotional and spiritual needs.
e. Ensure the coordination of the patient’s care.

Rationale: When a patient is initially interviewed and assessed, the nurse must complete a thorough analysis of the patient’s physical, emotional, spiritual, and psychomotor status. The nurse often works with unlicensed assistive personnel (UAP) to collect relevant
data on height and weight, intake and output, and vital signs. Nurses collaborate with other health care professionals to coordinate care. Interdisciplinary clinical rounds, which include physicians, registered nurses, physical therapists, occupational therapists, and dietitians, are often undertaken to identify priorities of care, discuss overlapping areas of treatment, and ensure coordination of care.

How well did you know this?
1
Not at all
2
3
4
5
Perfectly
22
Q

The nurse knows that professional nursing requires a commitment to which reasons for lifelong learning? (Select all that apply.)

a. Treatment modalities and technology continue to advance.
b. There are always new things to memorize and store in memory.
c. Nurses are expected to update and maintain competency.
d. Critical thinking is essential in nursing.
e. Nursing school gives the nurse all one needs to be competent.

A

a. Treatment modalities and technology continue to advance.
c. Nurses are expected to update and maintain competency.
d. Critical thinking is essential in nursing.

Rationale: Professional nursing requires a commitment to lifelong learning. Nurses must possess critical-thinking skills to maintain pace with ever-changing treatment modalities and technological advances. Outdated learning strategies that focus on remembering content must be replaced by a focus on understanding the rationales and outcomes. It is an expectation of professional practice that nurses
update and maintain their competency and knowledge base. The increasing complexity of health care and information technology make critical thinking essential in nursing. No longer is rote memorization and recall of content sufficient for the complex decisions and judgment required in professional nursing practice. Because knowledge and technology continue to expand for nursing professionals, the content learned in nursing school is not sufficient to maintain competence in nursing practice.

How well did you know this?
1
Not at all
2
3
4
5
Perfectly
23
Q

The nurse has been practicing for several years and has become an unofficial leader, with newer nurses asking for advice about patient care. They are amazed at how much the older nurse “thinks like a nurse.” To “think like a nurse,” the nurse must carry out
which actions? (Select all that apply.)

a. Be a nurse for several years.
b. Be able to apply knowledge in making clinical decisions.
c. Actively participate in the process.
d. Accept procedures that have been in place for years as right.
e. Develop a questioning attitude.

A

b. Be able to apply knowledge in making clinical decisions.
c. Actively participate in the process.
e. Develop a questioning attitude.

Rationale: Because nursing requires the application of knowledge to make clinical decisions and guide care, it involves active participation by the nurse. The application of knowledge requires the development of a questioning attitude. This process is sometimes referred to as thinking like a nurse. “Several years” is vague, and nurses develop critical thinking abilities at different rates. A questioning attitude does not accept doing things because they have been done that way for a long time.

How well did you know this?
1
Not at all
2
3
4
5
Perfectly
24
Q

The nurse identifies the nursing process as the foundation of professional nursing practice and can define it in which appropriate terms?

a. The framework that nurses use to provide care.
b. A complex process during which nurses think about their thinking.
c. The process that allows nurses to collect essential data.
d. Thinking like a nurse in developing plans of care.

A

a. The framework that nurses use to provide care.

Rationale: The nursing process is the foundation of professional nursing practice. It is the framework within which nurses provide care to patients in an organized and effective manner.
Paul describes critical thinking as a complex process during which individuals think about their thinking to provide clarity and increase precision and relevance in a specific situation while attempting to be fair and consistent.
Critical thinking using the nursing process allows nurses to collect essential patient data, articulate the specific needs of individual patients, and effectively communicate those needs, realistic goals, and customized interventions with members of the health care team.
Thinking like a nurse is facilitated by nurses using the nursing process in the development of individualized patient plans of care.

How well did you know this?
1
Not at all
2
3
4
5
Perfectly
25
Q

Since the nursing process is cyclic rather than linear, the nurse knows that as an individual patient’s condition changes the nurse should anticipate what concept?

a. The nurse’s thought processes do not have to vary.
b. Plans of care are easier to use and do not need modification.
c. The accuracy and effectiveness of thought processes must be considered.
d. Reflective thought is not necessary since issues tend to be repetitive.

A

c. The accuracy and effectiveness of thought processes must be considered.

Rationale: The nursing process is cyclic rather than linear. As an individual patient’s condition changes, so does the way a professional nurse thinks about that patient’s needs, forcing modification of earlier plans of care. At each step of the nursing process, nurses must
consider the accuracy and effectiveness of their thought process. This form of reflective thought is an essential aspect of critical thinking. The evolutionary nature of the nursing process allows nurses to adjust to changing patient needs. Plans of care must evolve as patients’ needs change.

How well did you know this?
1
Not at all
2
3
4
5
Perfectly
26
Q

The charge nurse is discussing a patient’s care plan during a team meeting. The team determines that the patient has not met the
goal of “ambulating to the nurse’s station twice a day” and decides to revise the plan. The nurse recognizes which characteristic of
the nursing process most represents this decision?

a. Organization
b. Dynamics
c. Adaptability
d. Outcome orientation

A

d. Outcome orientation

How well did you know this?
1
Not at all
2
3
4
5
Perfectly
27
Q

The nurse is caring for a patient who will be discharged home following surgical repair of a broken shoulder. The patient tells the nurse, “I don’t have anyone at home who can help me cook my meals. Is there something you can do?” When demonstrating the adaptability of the nursing process, the nurse should carry out which task?

a. Adjust the patient’s care plan so that nursing goals can be met.
b. Consult the care provider about extending the patient’s hospitalization.
c. Abandon the plan of care as not able to be done.
d. Contact the social worker about community services.

A

d. Contact the social worker about community services.

Rationale: The nursing process is adaptable for developing plans of care for individuals who are hospitalized or are receiving care in an outpatient, long-term care, or home setting. The nurse would adjust planning to contact the social worker for community resources so the patient can maintain as much independence as possible. The care plan focuses on the patient’s goals. The provider may or
may not be able to extend the hospital stay, but even if that were possible, the patient would not be able to stay until all function returned. The nurse does not simply abandon the care plan; the nurse looks for options and adaptations.

How well did you know this?
1
Not at all
2
3
4
5
Perfectly
28
Q

The nurse is completing the health history on a patient admitted for cardiac rehabilitation. The nurse recognizes that the health history is conducted in which step of the nursing process?

a. Assessment
b. Diagnosis
c. Implementation
d. Evaluation

A

a. Assessment

Rationale: During the assessment step, patient care data are gathered through observation, interviews, and physical assessment. In the diagnosis step, patient data are analyzed, validated, and clustered to identify patient problems. Each problem is then stated in
standardized language as a specific Nursing diagnosis to provide greater clarity and universal understanding by all care providers.
The implementation step includes initiating specific nursing interventions and treatments designed to help the patient achieve established goals or outcomes. In the evaluation step, the nurse determines whether the patient’s goals are met, examines the effectiveness of interventions, and decides whether the plan of care should be discontinued, continued, or revised.

How well did you know this?
1
Not at all
2
3
4
5
Perfectly
29
Q

The nurse is assisting a patient to bed when the patient says, “My chest hurts and my left arm feels numb. What’s wrong with me?”
What is the type and source of data obtained from the patient’s complaint?

a. Objective data from a primary source
b. Objective data from a secondary source
c. Subjective data from a primary source
d. Subjective data from a secondary source

A

c. Subjective data from a primary source

Rationale: Objective data consist of observable information that the nurse gathers on the basis of what can be seen, measured, or tested.
Subjective data are spoken. Primary data consist of information obtained directly from a patient. Secondary data are collected from family members, friends, other health care professionals, or written sources such as medical records and test results.

How well did you know this?
1
Not at all
2
3
4
5
Perfectly
30
Q

The nurse identifies which statement to be a correctly written Nursing diagnosis appropriate for a patient’s plan of care?

a. Ineffective airway clearance related to excessive secretions as evidenced by diminished breath sounds.
b. Imbalanced nutrition: less than body requirements.
c. Impaired physical mobility related to contractures.
d. Risk for suffocation related to smoking in bed as evidenced by absent breath sounds.

A

a. Ineffective airway clearance related to excessive secretions as evidenced by diminished breath sounds.

Rationale: There are three types of diagnoses: actual, risk, and opportunities for improvement. Actual diagnoses have three parts: problem, etiology, and signs/symptoms. Risk diagnoses include only the identified need and the risk factors. The Nursing diagnosis, imbalanced nutrition: less than body requirements, is missing the problem, etiology, and signs and symptoms. Impaired physical mobility is missing the evidence. Risk for suffocation should have only two parts: the potential problem and etiology. There are no signs and symptoms if the patient is at risk.

How well did you know this?
1
Not at all
2
3
4
5
Perfectly
31
Q

The nurse is admitting a patient experiencing chest discomfort and shortness of breath, who has a history of stroke. When the nurse documents the Nursing diagnosis “Risk for impaired mobility related to history of stroke,” the nurse knows which condition to be
the risk factor?

a. Stroke
b. History of stroke
c. Chest discomfort
d. Shortness of breath

A

b. History of stroke

Rationale: A two-part risk, Nursing diagnostic statement contains only: (1) the patient’s identified need or problem (i.e., NANDA-I Nursing diagnostic label) and (2) factors indicating vulnerability (i.e., risk factors). The risk factor is the history of stroke. The chest
discomfort and shortness of breath are symptoms of the current problems and would not be documented as potential or “risk” issues. “Stroke” would be the identified potential problem.

How well did you know this?
1
Not at all
2
3
4
5
Perfectly
32
Q

A patient with a congenital heart defect is admitted for further testing. The nurse observes the patient has increased shortness of breath and is restless. The nurse is demonstrating which phase of the nursing process?

a. Assessment
b. Planning
c. Implementation
d. Evaluation

A

a. Assessment

Rationale: During the assessment step, patient care data are gathered through observation, interviews, and physical assessment. During the planning step of the nursing process, the nurse prioritizes the Nursing diagnoses and identifies short- and long-term goals that are realistic, measurable, and patient-focused, with specific outcome identification for evaluation purposes. The implementation step includes initiating specific nursing interventions and treatments designed to help the patient achieve established goals or outcomes.
In the evaluation step, the nurse determines whether the patient’s goals are met, examines the effectiveness of interventions, and decides whether the plan of care should be discontinued, continued, or revised.

How well did you know this?
1
Not at all
2
3
4
5
Perfectly
33
Q

While the nurse is assisting with morning care, the patient has a tonic clonic seizure. The nurse immediately turns the patient to a side-lying position. The nurse is demonstrating which phase of the nursing process?

a. Assessment
b. Planning
c. Implementation
d. Evaluation

A

c. Implementation

Rationale: The implementation step includes initiating specific nursing interventions and treatments designed to help the patient achieve established goals or outcomes. During the assessment step, patient care data are gathered through observation, interviews, and
physical assessment. During the planning step of the nursing process, the nurse prioritizes the Nursing diagnoses and identifies short- and long-term goals that are realistic, measurable, and patient-focused, with specific outcome identification for evaluation
purposes. In the evaluation step, the nurse determines whether the patient’s goals are met, examines the effectiveness of interventions, and decides whether the plan of care should be discontinued, continued, or revised.

How well did you know this?
1
Not at all
2
3
4
5
Perfectly
34
Q

The nurse is caring for a patient diagnosed with Lyme disease. The patient tells the nurse, “My heart seems to be skipping some beats. My doctor told me to let the nurse know if this happens since it might be a complication of my disease.” The nurse auscultates the heart and confirms the palpitations. Which step of the nursing process does the nurse’s action demonstrate?

a. Assessment
b. Planning
c. Implementation
d. Evaluation

A

a. Assessment

Rationale: During the assessment step, patient care data are gathered through observation, interviews, and physical assessment. During the planning step of the nursing process, the nurse prioritizes the Nursing diagnoses and identifies short- and long-term goals that are realistic, measurable, and patient-focused, with specific outcome identification for evaluation purposes. The implementation step includes initiating specific nursing interventions and treatments designed to help the patient achieve established goals or outcomes.
In the evaluation step, the nurse determines whether the patient’s goals are met, examines the effectiveness of interventions, and decides whether the plan of care should be discontinued, continued, or revised.

How well did you know this?
1
Not at all
2
3
4
5
Perfectly
35
Q

In which step of the nursing process does the nurse prioritize the Nursing diagnoses and identify interventions to address the patient goals?

a. Assessment
b. Planning
c. Implementation
d. Evaluation

A

b. Planning

Rationale: During the planning step of the nursing process, the nurse prioritizes the Nursing diagnoses and identifies short- and long-term goals that are realistic, measurable, and patient focused, with specific outcome identification for evaluation purposes. During the assessment step, patient care data are gathered through observation, interviews, and physical assessment. The implementation step includes initiating specific nursing interventions and treatments designed to help the patient achieve established goals or outcomes.
In the evaluation step, the nurse determines whether the patient’s goals are met, examines the effectiveness of interventions, and decides whether the plan of care should be discontinued, continued, or revised.

How well did you know this?
1
Not at all
2
3
4
5
Perfectly
36
Q

The nurse writes a short-term goal for a patient scheduled for surgery in the morning and identifies which goal that contains all the necessary elements?

a. The patient will walk to the bathroom within 48 hours after surgery.
b. The patient will walk to the bathroom without experiencing shortness of breath within 48 hours after surgery.
c. The patient will walk to the bathroom without experiencing shortness of breath.
d. The patient will walk to the bathroom without experiencing shortness of breath after surgery.

A

b. The patient will walk to the bathroom without experiencing shortness of breath within 48 hours after surgery.

Rationale: All short- and long-term goals must be: (1) patient-focused, (2) realistic, and (3) measurable. For example, a patient-focused, realistic, and measurable short-term goal may be written for a patient with the Nursing diagnosis of Activity intolerance: The patient walks to the bathroom without experiencing shortness of breath within 48 hours after surgery.

How well did you know this?
1
Not at all
2
3
4
5
Perfectly
37
Q

A new community health nurse observes that a patient has generalized itching and a red rash after touching a latex glove. When the nurse asks the manager if there is a document written by the physician for this type of reaction, the nurse is referring to which concept?

a. Protocol
b. Clinical pathway
c. Standing order
d. Care map

A

c. Standing order

Rationale: Standing orders are written by physicians and list specific actions to be taken by a nurse or other health care provider when access to a physician is not possible or when care is common to a certain type of situation, such as what to do if a patient experiences chest pain or what actions to take after a colonoscopy. Protocols are written plans that can be generalized to groups of patients with the same or similar clinical needs that do not require a physician’s order. Health care agencies have established protocols outlining procedures for admitting patients or handling routine care situations. Clinical pathways, sometimes referred to as care pathways, care maps, or critical pathways, are multidisciplinary resources designed to guide patient care.

How well did you know this?
1
Not at all
2
3
4
5
Perfectly
38
Q

All nursing interventions that are implemented for patients must be documented or charted. The nurse knows that proper documentation of interventions leads to what positive outcome?

a. Proper documentation facilitates communication with all members of the health care team.
b. Proper documentation is only considered “legal” if documented in the paper chart.
c. Proper documentation prevents errors of omission and repetition of care.
d. Proper documentation does not directly measure goal achievement or outcomes.

A

a. Proper documentation facilitates communication with all members of the health care team.

Rationale: All nursing interventions that are implemented for patients must be documented or charted. In some cases, this may involve checking off an intervention in the patient’s EMR designed to track the effectiveness of specific interventions. Many health care agencies have special requirements for documenting interventions such as the use of physical restraints or pain protocols. Proper documentation of interventions facilitates communication with all members of the health care team and provides an essential legal record. Accurate charting helps to alleviate omissions and repetition of care although it cannot prevent them. Documentation also
allows nurses to evaluate the effectiveness of nursing interventions in meeting patient goals and outcomes, which is the final step in the nursing process.

How well did you know this?
1
Not at all
2
3
4
5
Perfectly
39
Q

The nurse makes the following entry on the patient’s care plan: “Goal not met. Patient refuses to walk and states, ‘I’m afraid of falling.’” The nurse should complete which next action?

a. Ignore the patient’s concern in evaluating goal attainment.
b. Document the patient’s unwillingness to continue the plan of care.
c. Continue the plan of care as originally agreed upon.
d. Modify the care plan in response to the patient’s condition and wishes.

A

d. Modify the care plan in response to the patient’s condition and wishes.

Rationale: Evaluation focuses on the patient and the patient’s response to nursing interventions and goal or outcome attainment. If a goal was not met, the care plan needs to be modified to avoid simply repeating the same actions. Ignoring the patient is not a therapeutic response. The nurse should respect the patient’s fear and assess further without simply documenting that the patient is unwilling.

How well did you know this?
1
Not at all
2
3
4
5
Perfectly
40
Q

The nurse is demonstrating how to correctly perform deep breathing and coughing exercises to a patient scheduled for back surgery. Which step of the nursing process is the nurse addressing?

a. Assessment
b. Diagnosis
c. Implementation
d. Evaluation

A

c. Implementation

Rationale: The implementation step includes initiating specific nursing interventions and treatments designed to help the patient achieve established goals or outcomes. During the assessment step, patient care data are gathered through observation, interviews, and
physical assessment. In the diagnosis step, patient data are analyzed, validated, and clustered to identify patient problems. Each problem is then stated in standardized language as a specific Nursing diagnosis to provide greater clarity and universal understanding by all care providers. In the evaluation step, the nurse determines whether the patient’s goals are met, examines the effectiveness of interventions, and decides whether the plan of care should be discontinued, continued, or revised.

How well did you know this?
1
Not at all
2
3
4
5
Perfectly
41
Q

The nurse develops a list of Nursing diagnoses for a patient receiving intravenous chemotherapy for breast cancer. The patient tells the nurse, “I understand that I will lose most of my hair. Will it grow back?” The nurse identifies which diagnosis will have the highest priority?

a. Disturbed body image
b. Nausea
c. Risk for bleeding
d. Imbalanced nutrition: less than body requirements

A

a. Disturbed body image

Rationale: Priority of Nursing diagnoses is determined by the patient’s preference as well as the severity of the symptoms. The patient is concerned about the loss of hair because this will affect body image. For the patient, this is a prime focus. It is possible that the
patient may experience nausea as a result of the chemotherapy drugs. The patient will not be able to eat properly if the nausea is not controlled thus decreasing nutritional intake. There is a potential for bleeding as a result of the low platelet count created by the
drugs. All of these must be addressed, but the primary diagnosis, in this case, would be body image.

How well did you know this?
1
Not at all
2
3
4
5
Perfectly
42
Q

The nurse is gathering data on a patient with acute bacterial pneumonia. The nurse recognizes that this is an example of which step of the nursing process?

a. Assessment
b. Planning
c. Implementation
d. Evaluation

A

a. Assessment

Rationale: During the assessment step, patient care data are gathered through observation, interviews, and physical assessment. In the diagnosis step, patient data are analyzed, validated, and clustered to identify patient problems. Each problem is then stated in
standardized language as a specific Nursing diagnosis to provide greater clarity and universal understanding by all care providers.
The implementation step includes initiating specific nursing interventions and treatments designed to help the patient achieve established goals or outcomes. In the evaluation step, the nurse determines whether the patient’s goals are met, examines the effectiveness of interventions, and decides whether the plan of care should be discontinued, continued, or revised.

How well did you know this?
1
Not at all
2
3
4
5
Perfectly
43
Q

The nurse knows which statements would be considered objective data? (Select all that apply.)

a. “I’m short of breath.”
b. “Blood pressure 90/68, apical pulse 102, skin pale and moist.”
c. “Lung sounds clear bilaterally, diminished in right lower lobe.”
d. “I feel weak all over when I exert myself.”
e. “My pain level is down to 2. It was 8.”

A

b. “Blood pressure 90/68, apical pulse 102, skin pale and moist.”
c. “Lung sounds clear bilaterally, diminished in right lower lobe.”

Rationale: Data collected from medical records, laboratory, and diagnostic test results, or physical assessments are objective. Objective data
(i.e., signs) consist of observable information that the nurse gathers on the basis of what can be seen, measured, or tested.
Subjective data (i.e., symptoms) are spoken. Patients’ feelings about a situation or comments about how they are feeling are examples of subjective data. Data shared by a source verbally are considered subjective. Subjective data may be difficult to validate
because they cannot be independently and objectively measured.

How well did you know this?
1
Not at all
2
3
4
5
Perfectly
44
Q

The nurse is attempting to develop Nursing diagnoses for a patient. The nurse understands that Nursing diagnoses have which characteristics? (Select all that apply.)

a. Nursing diagnoses identify actual or potential problems as well as responses to a problem.
b. Nursing diagnoses require naming patient problems using Nursing diagnostic labels.
c. Nursing diagnoses utilize objective data since subjective data are often inaccurate.
d. Nursing diagnoses include unvalidated data to determine an accurate and thorough diagnosis.
e. Nursing diagnoses are similar to medical diagnoses since they both are labels for diseases.

A

a. Nursing diagnoses identify actual or potential problems as well as responses to a problem.
b. Nursing diagnoses require naming patient problems using Nursing diagnostic labels.

Rationale: The Nursing diagnosis identifies an actual or potential problem or response to a problem. Accurate identification of Nursing diagnoses for patients results from carefully analyzing, validating, and clustering related patient subjective (symptoms) and
objective (signs) data. If data collection includes inaccurate or inadequate information or if data are not validated or clustered with related information, a patient may be misdiagnosed. Diagnosis in the nursing process requires naming patient problems using
Nursing diagnostic labels. Medical diagnoses are labels for diseases, whereas Nursing diagnoses describe a response to an actual or potential problem or life process.

How well did you know this?
1
Not at all
2
3
4
5
Perfectly
45
Q

The nurse recognizes that establishing short- and long-term goals to address Nursing diagnoses involve which actions? (Select all that apply.)

a. Discussion with the patient
b. Exclusion of family with making patient decisions
c. Collaboration with other members of health care team
d. Making the health care provider as the central figure
e. Coordination of care as collaborative care

A

a. Discussion with the patient
c. Collaboration with other members of health care team
e. Coordination of care as collaborative care

Rationale: Establishing short- and long-term goals to address Nursing diagnoses involves discussion with the patient and often requires collaboration with family members and other members of the health care team. Coordinated, team-based patient care is called
collaborative care. The patient’s health care team members may include several nurses: the primary care provider; medical or surgical specialists; respiratory therapists; a dietitian; a physical therapist; occupational, music, or art therapists; a spiritual adviser; and social workers. The patient’s primary nurse is often the central figure in coordinating collaborative care.

How well did you know this?
1
Not at all
2
3
4
5
Perfectly
46
Q

The nurse is caring for a patient with pneumonia, who is a retired soldier who served in World War II. With this information in mind, what should the nurse do in regarding this patient?

a. Shake the patient’s hand and allow the patient time to “warm up.”
b. Expect the patient to be optimistic and question everything.
c. Allow the patient to multitask and talk in short “sound bites.”
d. Understand that the patient is probably technologically literate.

A

a. Shake the patient’s hand and allow the patient time to “warm-up.”

Rationale: Establishing rapport is paramount to gaining the trust of the patient. The nurse should consider the patient’s generational cohort, which may influence behavior, and willingness to share personal information during the interview process.
Veterans (born before 1945) respect authority; are detail-oriented; communicate in a discrete, formal, respectful way; may be slow to warm up; value family and community and accept physical touch as an effective form of therapeutic communication.
Baby Boomers (born 1946 to 1964) are optimistic, relationship-oriented, and communicate by using open or direct speech, using body language, and answering questions thoroughly. They expect detailed information, question everything, and value success. Generation X members (born 1965 to 1976) are informal; are technology immigrants; multitask; communicate in a blunt or direct, factual, and informal style; may talk in short sound bites; share information frequently; and value time.
Millennials, also called Generation Y (born 1977 to 1994) are flexible; are technologically literate or are technology natives; multitask; communicate by using action verbs and humor; may be brief in the form of texting or e-mail exchanges; like personal attention; and value individuality.
Individuals from Generation Z (born 1995 to 2012) are digitally connected, value group work, want immediate feedback, are accepting of others, value honesty, and family, and are entrepreneurial.

How well did you know this?
1
Not at all
2
3
4
5
Perfectly
47
Q

The patient interview consists of three phases: orientation (introductory), working, and termination. Each phase contributes to the development of trust and engagement between the nurse and the patient. During the orientation phase of a patient interview, the
nurse carries out what action?

a. Obtain demographic data using open-ended questions.
b. Establish the name by which the patient prefers to be addressed.
c. Gather general information using closed-ended questions.
d. Stand by the bedside to ask the needed questions.

A

b. Establish the name by which the patient prefers to be addressed.

Rationale: The patient interview consists of three phases: orientation (introductory), working, and termination. Each phase contributes to the development of trust and engagement between the nurse and the patient.
During the orientation phase of the interview, the nurse should establish the name by which the patient prefers to be addressed. Some individuals prefer formal titles of respect (e.g., Dr., Mr., Ms., Professor) and the use of surnames, whereas others are comfortable with less formality.
How a patient is addressed is the patient’s choice. Demographic data should be collected by asking focused or closed-ended questions.
More general information can be gathered by open-ended communication techniques. When feasible, the nurse and the patient should be seated at eye level with
each other. In this way, the interaction between the nurse and the patient is horizontal instead of vertical. Standing over someone implies control, power, and authority.
The implication of power can result in less-than-optimal data collection and a potential conflict as the patient strives to regain control over the situation.

How well did you know this?
1
Not at all
2
3
4
5
Perfectly
48
Q

A nurse is conducting a health interview on a newly admitted patient. To establish a trusting relationship with the patient, the nurse carries out which action?

a. Avoid eye contact to appear less threatening.
b. Demonstrate professionalism by not smiling.
c. Sit close and leans in slightly toward the patient.
d. Speaks in a slow rate of speech and low tone.

A

c. Sit close and leans in slightly toward the patient.

Rationale: Nonverbal behaviors of the nurse can influence the information obtained from the patient. Negative nonverbal cues such as distracting gestures (e.g., tapping a pen, swinging a foot, looking at a watch), inappropriate facial expressions, and lack of eye contact communicate disinterest.
To establish a trusting relationship with the patient before the physical examination is conducted, the nurse should communicate professionally, sit close and lean in slightly toward the patient, listen attentively and demonstrate appropriate eye contact, smile, and use a moderate rate of speech and tone of voice.

How well did you know this?
1
Not at all
2
3
4
5
Perfectly
49
Q

The nurse is assigned the admission health history and physical for a patient diagnosed with a fever of unknown etiology. The patient tells the nurse, “I just don’t feel good. I’m so hot and I feel sick to my stomach. Can you ask me those questions later?”
What would be the best response by the nurse?

a. “It will not take too long. I can hurry.”
b. “We need the information to complete your admission paperwork.”
c. “I will come back in a few minutes and we can start over.”
d. “Let me see if you can have something for the nausea and then talk later.”

A

d. “Let me see if you can have something for the nausea and then talk later.”

Rationale: If a patient being admitted to the hospital is too ill to interact for an extended period, the interview can be broken into smaller segments. Interviews with patients already hospitalized or established in the health care system are less extensive and more focused
on newly identified patient concerns or problems. Ensuring that the patient is comfortable and relaxed is a priority and often takes prior thought and planning by the nurse.

How well did you know this?
1
Not at all
2
3
4
5
Perfectly
50
Q

The nurse is using a stethoscope to assess a patient’s cardiac status. Which assessment technique is the nurse using?

a. Inspection
b. Percussion
c. Palpation
d. Auscultation

A

d. Auscultation

Rationale: Auscultation is a technique of listening with the assistance of a stethoscope to sounds made by organs or systems such as the heart, blood vessels, lungs, and abdominal cavity.
Inspection involves the use of vision, hearing, and smell to closely scrutinize physical characteristics of a whole person and individual body systems.
Percussion involves tapping the patient’s skin with short, sharp strokes that cause a vibration to travel through the skin and to the upper layers of the underlying structures. Vibration is reflected
by the tissues, and the character of the sound heard depends on the density of the structures that reflect the sound.
Palpation uses touch to assess body organs and skin texture, temperature, moisture, turgor, tenderness, and thickness.

How well did you know this?
1
Not at all
2
3
4
5
Perfectly
51
Q

The nurse is performing an assessment of a patient’s right kidney. The nurse bluntly strikes the area of the costovertebral angle while observing the patient’s reaction. Which assessment technique is the nurse using?

a. Inspection
b. Percussion
c. Palpation
d. Auscultation

A

b. Percussion

Rationale: Percussion involves tapping the patient’s skin with short, sharp strokes that cause a vibration to travel through the skin and to the upper layers of the underlying structures.
Inspection involves the use of vision, hearing, and smell to closely scrutinize physical characteristics of a whole person and individual body systems.
Palpation uses touch to assess body organs and skin texture, temperature, moisture, turgor, tenderness, and thickness.
Auscultation is a technique of listening with the assistance of a stethoscope to sounds made by organs or systems such as the heart, blood vessels, lungs, and abdominal cavity. Vibration is reflected by the tissues, and the character of the sound heard depends on the density of the structures that reflect the sound.

How well did you know this?
1
Not at all
2
3
4
5
Perfectly
52
Q

The nurse is performing a physical exam on a patient diagnosed with liver failure resulting from chronic alcoholism. The nurse notes that the abdomen is swollen and decides to assess for abdominal skin tenderness and temperature. Which technique would the nurse use to collect this data?

a. Inspection
b. Percussion
c. Palpation
d. Auscultation

A

c. Palpation

Rationale: Palpation uses touch to assess body organs and skin texture, temperature, moisture, turgor, tenderness, and thickness.
Inspection involves the use of vision, hearing, and smell to closely scrutinize physical characteristics of a whole person and individual body systems.
Percussion involves tapping the patient’s skin with short, sharp strokes that cause a vibration to travel through the skin and to the upper layers of the underlying structures.
Auscultation is a technique of listening with the assistance of a stethoscope to sounds made by organs or systems such as the heart, blood vessels, lungs, and abdominal cavity. Vibration is reflected by the
tissues and the character of the sound heard depends on the density of the structures that reflect the sound.

How well did you know this?
1
Not at all
2
3
4
5
Perfectly
53
Q

The triage nurse in a hospital emergency department is determining the order of care for several patients. Which patient would the nurse consider as having the highest priority?

a. A 68-year-old patient suffering from dehydration and disorientation
b. A 14-year-old patient having respiratory distress and increasing anxiety
c. A 46-year-old patient with multiple cuts and abrasions to the upper extremities
d. A 38-year-old patient with a broken right hip and in severe pain

A

b. A 14-year-old patient having respiratory distress and increasing anxiety

Rationale: Triage, a form of emergency assessment, is the classification of patients according to treatment priority. Patients are categorized by the urgency of their condition. Most emergency departments use a five-tier triage system. The five-tier system classifies patients by levels numbered 1 through 5.
Level 1 is considered critical: life-threatening conditions require immediate and continuous care such as severe trauma, cardiac arrest, respiratory distress, seizure, or shock.
Level 2 emergencies can be imminently life-threatening conditions requiring care within 30 minutes, such as chest pain or major fractures, with severe pain.
Level 3 is considered urgent: potentially life-threatening conditions that require care within 30 to 60 minutes, such as minor fractures, lacerations, and
dehydration.
Level 4 is considered semi-urgent, stable health conditions that require care within 60 to 120 minutes, such as a twisted ankle.
Level 5 conditions are non-urgent and lower risk such as cold symptoms.

How well did you know this?
1
Not at all
2
3
4
5
Perfectly
54
Q

The morning nurse is assigned to care for a patient admitted during the night with rectal bleeding. When making rounds, the nurse observes that the patient’s face is ashen in color and the skin is cool and clammy. The nurse auscultates the patient’s heart and lungs. Which category of physical assessment is the basis for the nurse’s response?

a. Emergency assessment
b. Focused assessment
c. Complete assessment
d. Initial comprehensive

A

a. Emergency assessment

Rationale: Emergency assessment is a physical examination done when time is a factor, treatment must begin immediately, or priorities for care need to be established in a few seconds or minutes. Attention is paid to the patient’s airway, breathing, and circulation. Other concerns in the emergent setting are noticeable deformities such as compound fractures, contusions, abrasions, puncture wounds, burns, tenderness, lacerations, bleeding, and swelling. During an emergency, the nurse may never have time to do a complete assessment and may work to stabilize one body system at a time.
A focused or clinical assessment is a brief individualized physical examination conducted at the beginning of an acute care setting work shift to establish current patient status or during ongoing patient encounters in response to a specific patient concern. A focused assessment may be conducted when signs indicate a change in a patient’s condition or the development of a new complication.
A comprehensive or complete assessment includes a thorough interview, health history, review of systems, and extensive physical head-to-toe assessment, including evaluation of cranial nerves and sensory organs, such as with sight and hearing testing.
A complete physical examination may be conducted on admission to a hospital, during an annual physical at the office of a physician or nurse practitioner, or on initial interaction with a specialist.

How well did you know this?
1
Not at all
2
3
4
5
Perfectly
55
Q

The nurse is performing her initial assessment of the day when she notices that the patient has a facial droop that was not present yesterday and that was not reported in the hand-off report from the night nurse. The nurse proceeds to assess the neurologic status
of the patient and knows this to be which type of assessment?

a. Emergency assessment
b. Focused assessment
c. Complete physical examination
d. Comprehensive assessment

A

b. Focused assessment

Rationale: A focused or clinical assessment is a brief individualized physical examination conducted at the beginning of an acute care setting work shift to establish current patient status or during ongoing patient encounters in response to a specific patient concern.
A focused assessment may be conducted when signs indicate a change in a patient’s condition or the development of a new complication. Emergency assessment is a physical examination done when time is a factor, treatment must begin immediately, or
priorities for care need to be established in a few seconds or minutes. Attention is paid to the patient’s airway, breathing, and circulation. Other concerns in the emergent setting are noticeable deformities such as compound fractures, contusions, abrasions,
puncture wounds, burns, tenderness, lacerations, bleeding, and swelling.
A comprehensive or complete assessment includes a thorough interview, health history, review of systems, and extensive physical head-to-toe assessment, including evaluation of cranial nerves and sensory organs, such as with sight and hearing testing.
A complete physical examination may be conducted on
admission to a hospital, during an annual physical at the office of a physician or nurse practitioner, or on initial interaction with a specialist

How well did you know this?
1
Not at all
2
3
4
5
Perfectly
56
Q

The nurse is documenting data collected during a health assessment interview. Which statement by the nurse indicates subjective data?

a. “My last bowel movement was 4 days ago.”
b. Abdomen distended; firm and tender.
c. Dark-colored; hard pellet-shaped stool.
d. Color pink. Skin warm and dry. No sign of discomfort.

A

a. “My last bowel movement was 4 days ago.”

Rationale: Subjective data are spoken information or symptoms that cannot be authenticated. Subjective data usually are gathered during the interview process if patients are well enough to describe their symptoms. Objective data, also referred to as signs, can be measured or observed. The nurse’s senses of sight, hearing, touch, and smell are used to collect objective data. Objective assessment data are acquired through observation, physical examination, and analysis of laboratory and diagnostic test results.

How well did you know this?
1
Not at all
2
3
4
5
Perfectly
57
Q

A patient is transported to the emergency department from a local skilled nursing facility and admitted for a bacterial blood infection. The nurse reviews the transferring physician notes, which indicate that the patient has dementia. The nurse contacts the
patient’s son for additional health history information. Information provided by the son would be considered which type of data?

a. Primary, objective data
b. Primary, subjective data
c. Secondary, objective data
d. Secondary, subjective data

A

d. Secondary, subjective data

Rationale: Subjective data are spoken information or symptoms that cannot be authenticated. Subjective data usually are gathered during the interview process if patients are well enough to describe their symptoms. Family members, friends, and other members of the
health care team can contribute valid secondary, subjective data.
Objective data also referred to as signs, can be measured or observed. The nurse’s senses of sight, hearing, touch, and smell are used to collect objective data.
Objective assessment data are acquired through observation, physical examination, and analysis of laboratory and diagnostic test results. Primary data come directly from the patient.

How well did you know this?
1
Not at all
2
3
4
5
Perfectly
58
Q

The nurse is monitoring the blood sugar results of a patient receiving an intravenous nutritional supplement. The patient tells the nurse, “I have never had sugar problems before. My doctor says it is because I am getting this IV.” These types of data are
considered to be which type?

a. Primary, objective data
b. Primary, subjective data
c. Secondary, objective data
d. Secondary, subjective data

A

b. Primary, subjective data

Rationale: Primary data come directly from the patient. Subjective data are spoken information or symptoms that cannot be authenticated.
Subjective data usually are gathered during the interview process if patients are well enough to describe their symptoms. Family members, friends, and other members of the health care team can contribute valid secondary, subjective data.
Objective data also referred to as signs, can be measured or observed. The nurse’s senses of sight, hearing, touch, and smell are used to collect
objective data. Objective assessment data are acquired through observation, physical examination, and analysis of laboratory and diagnostic test results.

How well did you know this?
1
Not at all
2
3
4
5
Perfectly
59
Q

The unlicensed nursing assistive person (UAP) reports to the nurse that a patient is crying during a comedy show on television.
What would be the best response by the nurse?

a. “Maybe the patient doesn’t think the show is funny.”
b. “Don’t worry about it. The patient’s daughter says this is normal.”
c. “I will go visit her right away and see what is going on.”
d. “Just document what you observe in your notes.”

A

c. “I will go visit her right away and see what is going on.”

Rationale: Validating data is making sure that the data are accurate. As patient information is collected, consistency between subjective and objective data must be confirmed. Confirming the validity of collected data often requires verbally checking with the patient to see whether the assumptions or conclusions at which the nurse arrived are correct. Crying, a disheveled appearance and lack of eye contact may be cues of depression. However, conclusions about the underlying cause of the patient’s actions cannot be assumed.
All cues need to be interpreted and validated to verify the data’s accuracy. The nurse cannot assume that this is normal behavior nor ignore the problem by making a joke. The nurse has the responsibility to attempt to determine the real reason for the crying episode.

How well did you know this?
1
Not at all
2
3
4
5
Perfectly
60
Q

A patient with moderate lower back pain tells the nurse, “My urine smells awful and is as dark as my glass of tea.” Which action by the nurse will assist in validating the patient’s concern?

a. Ask the patient to describe the back pain.
b. Review the lab results of the most recent urinalysis.
c. Request the nursing assistant to obtain a set of vital signs.
d. Check the patient’s history for urinary tract infections.

A

b. Review the lab results of the most recent urinalysis.

Rationale: As patient information is collected, consistency between subjective and objective data must be confirmed. Sometimes, the nurse
can use laboratory and diagnostic test results to validate the subjective data. In this case, checking the urinalysis for congruency with the patient’s subjective data will validate the patient’s statements. Obtaining a set of vital signs, reviewing the patient’s history, and exploring the patient’s pain are appropriate actions but cannot validate the current problem.

How well did you know this?
1
Not at all
2
3
4
5
Perfectly
61
Q

The nurse is attempting to get the patient to sign the operative consent. When asked if the health care provider explained the procedure to the patient, the patient replies “Not much.” What action will the nurse take next?

a. Develop a comprehensive teaching plan related to the surgical procedure.
b. Ask the patient what information the surgeon has explained about the surgery.
c. Contact the surgeon to clarify information given to the patient.
d. Focus on post-operative exercises and home-care following surgery.

A

b. Ask the patient what information the surgeon has explained about the surgery.

Rationale: Careful observation and attention to detail help the nurse to notice subtle cues and recognize how best to validate and interpret patient data. The nurse must be careful not to make false assumptions or generalizations regarding the patient’s responses to the health concern. The nurse is correct to ask the patient about the upcoming surgical procedure instead of assuming that the patient has limited knowledge. This is the nurse’s best action to determine what the surgeon said to the patient. Developing a
comprehensive teaching plan is not necessary until further clarification is obtained. Focusing on postoperative treatment plans is important but not the priority at this time. It is not appropriate to contact the surgeon unless the patient demonstrates an actual
knowledge deficit.

How well did you know this?
1
Not at all
2
3
4
5
Perfectly
62
Q

After the patient’s data are collected, validated, and interpreted, the nurse organizes the information in a framework (format) that facilitates access by all members of the health care team. What is the framework that provides the most holistic view of the
patient’s condition?

a. Head-to-toe pattern
b. Functional Health Patterns
c. Cephalic-caudal pattern
d. Body systems model

A

b. Functional Health Patterns

Rationale: Marjory Gordon developed the Functional Health Patterns to help nurses focus on patient strengths and related but sometimes overlooked data relationships.
This method of organizing patient data is a more holistic approach than the others because it includes data such as values, beliefs, and roles in addition to physical data.
Organizing assessment data in a head-to-toe (cephalic-caudal) pattern ensures that all areas of concern are addressed as the nurse performs an assessment covering the entire body.
The body systems model organizes data on the basis of each system of the body: integumentary, respiratory, cardiovascular, nervous, reproductive, musculoskeletal, gastrointestinal, genitourinary, and immune systems. It follows a sequence similar to the medical model for physical examination.
The body systems model for data organization tends to focus on the physical aspects of a patient’s condition rather than a more holistic view.

How well did you know this?
1
Not at all
2
3
4
5
Perfectly
63
Q

The nurse knows what should be included in an in-depth health history? (Select all that apply.)

a. Demographic data
b. Patient’s allergies
c. Family history of diseases
d. Patient’s health promotion practices
e. Patient’s history of illness and surgery

A

a. Demographic data
b. Patient’s allergies
c. Family history of diseases
d. Patient’s health promotion practices
e. Patient’s history of illness and surgery

Rationale: An in-depth health history includes all pertinent information that can guide the development of a patient-centered plan of care. The health history includes demographic data, which are collected during the orientation phase of the interview; a patient’s chief
complaint or reason for seeking health care; history of current and past illnesses and surgery; allergies; medications; adverse reactions to medications; medical history; family and social history; and health promotion practices. Because a patient’s health history is continuously evolving, the data collection is ongoing, progressive, and methodical.

How well did you know this?
1
Not at all
2
3
4
5
Perfectly
64
Q

The nurse is admitting a patient for uncontrolled diabetes mellitus. The nurse suspects that the patient could benefit from diabetic teaching. What actions by the nurse will assist in validating this suspicion? (Select all that apply.)

a. Determine the patient’s cognitive ability and potential language barriers.
b. Gather information about what the patient already knows about diabetes.
c. Have the patient demonstrate checking a blood glucose level.
d. Formulate the patient’s plan of care using a standard protocol.
e. Prepare to teach the patient using materials written at a third-grade level.

A

b. Gather information about what the patient already knows about diabetes.
c. Have the patient demonstrate checking a blood glucose level.

Rationale: Data that would validate the nurse’s suspicion that the patient needs further education include determining what the patient already
knows about diabetes and having the patient demonstrate the technique of blood glucose monitoring. If the nurse is correct, further
education is needed. Before further education can occur however; the nurse should determine if the patient has cognitive difficulties or a language barrier which would all contribute to an individualized plan of care. Reading material should typically be written at a
fifth-grade level, but the nurse should not assume the patient needs third-grade level material.

How well did you know this?
1
Not at all
2
3
4
5
Perfectly
65
Q

The nurse is preparing to begin a physical examination for a patient with open lesions on the lower extremities. Which would the nurse evaluate during the physical assessment? (Select all that apply.)

a. Blood test results
b. X-ray results
c. Recent vital signs
d. Patient’s health history
e. Subjective data

A

a. Blood test results
b. X-ray results
c. Recent vital signs

Rationale: On completion of the patient interview, health history, and review of systems, the nurse begins the physical assessment. During the physical assessment, the nurse collects objective data. If diagnostic tests, such as blood tests or x-rays, were ordered before the patient was seen, the results are reviewed by the nurse. Vital signs are taken and recorded at the beginning of the physical
examination.

How well did you know this?
1
Not at all
2
3
4
5
Perfectly
66
Q

The charge nurse is planning vital sign assignments for the unlicensed assistive personnel (UAP) on a busy medical-surgical unit. Which patients are appropriate for the UAP to obtain vital signs? (Select all that apply.)

a. A 28-year old patient scheduled to be discharged home today
b. A 49-year-old patient with stable chronic lung disease
c. A 78-year-old patient with recent onset of rectal bleeding
d. A 35-year-old patient waiting for transfer to a rehabilitation center
e. A 40-year-old patient being admitted from the emergency department

A

a. A 28-year old patient scheduled to be discharged home today
b. A 49-year-old patient with stable chronic lung disease
d. A 35-year-old patient waiting for transfer to a rehabilitation center

Rationale: Routine assessment of vital signs of a patient who is stable may be delegated to licensed practical or licensed vocational nurses (LPNs/LVNs) or qualified UAP. Initial and ongoing assessment of patients requiring critical care or who are unstable cannot be
delegated to UAPs. The patient with rectal bleeding may need critical care, and a new admission needs to be assessed by an RN.
Stable patients such as patients with stable lung disease or awaiting discharge or transfer can be delegated to UAP.

How well did you know this?
1
Not at all
2
3
4
5
Perfectly
67
Q

The nurse identifies which examples listed indicate objective data? (Select all that apply.)

a. Respirations—24 breaths/min
b. Platelet count—350,000 mm3
c. Wound size—3 cm × 2 cm
d. Temperature—98.4 °F (36.8 °C)
e. Reports severe abdominal pain

A

a. Respirations—24 breaths/min
b. Platelet count—350,000 mm3
c. Wound size—3 cm × 2 cm
d. Temperature—98.4 °F (36.8 °C)

Rationale: Objective data, also referred to as signs, can be measured or observed. The nurse’s senses of sight, hearing, touch, and smell are used to collect objective data.
Objective assessment data are acquired through observation, physical examination, and analysis of
laboratory and diagnostic test results. Subjective data are spoken information or symptoms that cannot be authenticated.
Subjective data usually are gathered during the interview process if patients are well enough to describe their symptoms.

How well did you know this?
1
Not at all
2
3
4
5
Perfectly
68
Q

Patient-centered care requires the nurse to complete which actions? (Select all that apply.)

a. Have an understanding of patient preferences.
b. Be aware of family values.
c. Recognize the patient’s expectations.
d. Base conclusions on the nurse’s personal experiences.
e. Provide care in a standardized manner.

A

a. Have an understanding of patient preferences.
b. Be aware of family values.
c. Recognize the patient’s expectations.

Rationale: Patient-centered care requires the nurse to understand patient and family preferences and values. Nurses must recognize patients’ expectations for care and provide care with respect for the diversity of human experience.
While interpreting data, the nurse must be careful to avoid inaccurate inferences (i.e., conclusions) based on the nurse’s personal preferences, past experiences,
generalizations, or outdated and inaccurate health care information.

How well did you know this?
1
Not at all
2
3
4
5
Perfectly
69
Q

The nurse completes a health and physical assessment on a patient admitted with a fractured pelvis. Which task would the nurse do next?

a. Analyze and cluster the assessment information.
b. Formulate a Nursing diagnosis addressing actual issues.
c. Determine the need for potential Nursing diagnoses.
d. Create health promotion diagnoses for the patient.

A

a. Analyze and cluster the assessment information.

Rationale: Nursing diagnosis is the second step of the nursing process. Formulation of nursing diagnoses follows patient data collection and involves the analysis and clustering of related assessment information. Actual nursing diagnoses identify existing problems or
concerns of a patient. Risk nursing diagnoses apply when there is an increased potential or vulnerability for a patient to develop a problem or complication. Health-promotion nursing diagnoses are used in situations in which patients express interest in improving
their health status through a positive change in behavior. The analysis of information is required to determine nursing diagnoses.

How well did you know this?
1
Not at all
2
3
4
5
Perfectly
70
Q

A group of patients in a community center attend a nursing-led information session on the risks of contracting tuberculosis. After the presentation, several patients ask the nurse for additional web-based resources regarding the lung disease. Which type of nursing diagnosis would the nurse choose for the community care plan?

a. Risk
b. Actual
c. Health-promotion
d. Potential

A

c. Health-promotion

Rationale: Health-promotion nursing diagnoses are used in situations in which patients express interest in improving their health status through a positive change in behavior. Although most nursing diagnoses are used for individual patients, nursing diagnosis
taxonomy can be applied to families, groups of individuals, and communities.
Actual nursing diagnoses identify existing problems or concerns of a patient. Risk (potential) nursing diagnoses apply when there is an increased potential or vulnerability for a patient
to develop a problem or complication.

71
Q

A nurse completes a care plan for an assigned patient diagnosed with an inflammation of the pericardium. Which diagnosis written on the plan indicates a need for further instruction on using the nursing process?

a. Pericarditis
b. Acute pain
c. Anxiety
d. Activity intolerance

A

a. Pericarditis

Rationale: Whereas medical diagnoses identify and label medical (physical and psychological) illnesses, nursing diagnoses are much broader in and consider a patient’s response to medical diagnoses and life situations.
The underlying etiology, or cause of a patient’s concern or situation, rather than a medical diagnosis, should be used as a related factor when writing an ICNP nursing diagnosis statement and can be included in the list of supporting data in the EMR.
Pericarditis is a medical diagnosis defined as an inflammation of the pericardium. Pain, anxiety and intolerance to activity are all possible patient responses to the medical condition of pericarditis.

72
Q

A patient is receiving an experimental drug for leukemia. The nurse is worried that the drug may cause a reduction in platelets leading to intestinal tract bleeding. Which type of nursing diagnosis does the nurse use to address this concern?

a. Risk
b. Actual
c. Health-promotion
d. Medical diagnosis

A

a. Risk

Rationale: The three types of nursing diagnostic statements are actual, risk, and health promotion. Determining which type is needed for each
patient can be challenging. Risk (potential) Nursing diagnoses apply when there is an increased potential or vulnerability for a patient to develop a problem or complication.
Actual Nursing diagnoses identify existing problems or concerns of a patient.
Health-promotion Nursing diagnoses are used in situations in which patients express interest in improving their health status through a positive change in behavior.
Although most nursing diagnoses are used for individual patients, nursing diagnosis taxonomy can be applied to families, groups of individuals, and communities. Medical diagnoses identify and label medical (physical and psychological) illnesses.

73
Q

The nurse is writing the care plan for a patient admitted to the hospital for complications associated with muscular dystrophy.
Which Nursing diagnoses written on the care plan indicate a need for further instruction in constructing the diagnostic statement?

a. Constipation related to immobility as manifested patient passing hard, dry stool with difficulty
b. Activity intolerance related to weakness as evidenced by verbal report of fatigue.
c. Impaired self-feeding related to fatigue as manifested by inability to open containers and bring food to the mouth.
d. Impaired airway clearance related to muscle weakness.

A

d. Impaired airway clearance related to muscle weakness.

Rationale: Each type of Nursing diagnostic statement contains sections or parts.
Actual Nursing diagnostic statements are written with three parts: a diagnosis label, related factors, and defining characteristics. Risk Nursing diagnoses have two segments: a diagnosis label and risk factors.
Health-promotion Nursing diagnoses are written with only two sections: the diagnosis label and defining characteristics. The impaired airway clearance label is missing the defining characteristics.

74
Q

Nursing students are analyzing the following Nursing diagnostic statement during a study group session. Acute pain related to pressure on lumbar spinal nerves as evidenced by a pain level of 9, patient verbalizations of pain, and grimacing when walking.
The students would be correct if they stated which response to be the etiology of the patient’s problem?

a. Patient verbalizations of pain
b. Acute pain
c. Pressure on lumbar spinal nerves
d. Grimacing when walking

A

c. Pressure on lumbar spinal nerves

Rationale: The second part of the Nursing diagnosis consists of related factors (for actual Nursing diagnoses) and risk factors (for risk Nursing
diagnoses). Related factors are the underlying cause or etiology of a patient’s problem. Risk factors are environmental, physical, psychological, or situational concerns that increase a patient’s vulnerability to a potential problem or concern. In this case, the acute
pain is being caused by pressure on the lumbar spinal nerves.

75
Q

The nurse is caring for a patient diagnosed with blood clots in the right lower extremity. The admitting provider orders bed rest. The patient tells the nurse, “I usually exercise three times a week. It helps me go to the bathroom.” The nurse determines that the
patient may have difficulty with bowel movements. Which Nursing diagnosis statement accurately reflects the nurse’s concern?

a. Constipation related to bed rest as manifested by hard, dry stools.
b. Constipation resulting from reduced peripheral circulation manifested by patient’s anxiety.
c. Risk for constipation related to immobility as manifested by verbal complaint.
d. Risk for constipation related to insufficient physical activity.

A

d. Risk for constipation related to insufficient physical activity.

Rationale: Each type of Nursing diagnostic statement contains sections or parts. Actual Nursing diagnostic statements are written with three parts: a diagnosis label, related factors, and defining characteristics. Risk Nursing diagnoses have two segments: a diagnosis label
and risk factors. Health-promotion Nursing diagnoses are written with only two sections: the diagnosis label and defining characteristics. There are no data suggesting the patient is constipated at this time.

76
Q

The nursing student is reviewing the components of a Nursing diagnosis. Which statement made by the student indicates correct understanding of a health-promotion diagnostic statement?

a. “The defining characteristics will include the patient’s willingness to get better.”
b. “The risk factors are only psychological in nature, not physical.”
c. “The health-promotion diagnostic statement is composed of three parts.”
d. “An example of a health-promotion label is ineffective community coping.”

A

a. “The defining characteristics will include the patient’s willingness to get better.”

Rationale: The three types of Nursing diagnostic statements are actual, risk, and health promotion. Determining which type is needed for each patient can be challenging.
Health-promotion Nursing diagnoses are used in situations in which patients express interest in improving their health status through a positive change in behavior. The second part of the Nursing diagnosis consists of related factors (for actual Nursing diagnoses) and risk factors (for risk Nursing diagnoses). Related factors are the underlying cause or
etiology of a patient’s problem.
Risk factors are environmental, physical, psychological, or situational. Health-promotion Nursing diagnoses are written with only two sections: the diagnosis label and defining characteristics.
Actual diagnoses describe the person, family, or community’s response to a health condition or life process that already has occurred. “Ineffective community coping” would be an example of an actual problem.

77
Q

The nurse is reviewing assessment findings on a patient admitted with an extremely slow heart rate in preparation to write a care plan. The patient complains of dizziness, shortness of breath, chest pain, and fainting spells. Vital signs are blood pressure of 98/60 mm Hg and pulse of 52 beats/min. Oxygen saturation is 88%. Which action does the nurse perform next?

a. Exclude all subjective data in favor of objective data.
b. Focus on data gathered during the physical assessment.
c. Evaluate the data looking for patterns and related data.
d. Dismiss family members input as “hearsay.”

A

c. Evaluate the data looking for patterns and related data.

Rationale: After collecting and reviewing all of the assessment data, the nurse looks for patterns and related data to support specific Nursing
diagnoses. This process is referred to as clustering data.
Clustering involves organizing patient assessment data into groupings with similar underlying causes. All patient information should be considered as potentially contributing to the identification of diagnostic labels. This information includes subjective and objective data collected through physical assessment of the patient,
interview of the patient and family members, and laboratory and diagnostic test results, including x-rays, physicians’ orders, and documentation from health care providers.
Verifying specific Nursing diagnoses for a patient or situation follows accurate analysis and clustering of data.

78
Q

The nurse is admitting a patient with severe dehydration. Assessment data reveal a decreased blood pressure, an increased pulse rate, and a low circulating blood volume. The student observes that the patient is confused and restless. Which patient information would the nurse consider as a contributing factor when choosing the Nursing diagnostic label?

a. Blood pressure, pulse rate
b. Blood pressure, pulse rate, blood volume
c. Blood pressure, pulse rate, blood volume, mental status
d. Blood pressure, pulse rate, blood volume, mental status, dehydration

A

d. Blood pressure, pulse rate, blood volume, mental status, dehydration

Rationale: All patient information should be considered as potentially contributing to the identification of diagnostic labels. This information includes subjective and objective data collected through physical assessment of the patient, interview of the patient and family members, and laboratory and diagnostic test results, including x-rays, physicians’ orders, and documentation from health care providers. Verifying specific Nursing diagnoses for a particular patient or situation follows accurate analysis and clustering of data.

79
Q

The nurse is reviewing data obtained through the health history interview and physical assessment of an assigned patient. Data collected include dry skin, brittle nails, weight gain, thinning hair, constipation, prolonged menstruation, and the patient’s
complaints of feeling tired and cold. The nurse recognizes which statement represents an appropriate data cluster?

a. Prolonged menstruation, constipation
b. Dry skin, brittle nails, weight gain
c. Tired, cold, thinning hair
d. Constipation, weight gain

A

d. Constipation, weight gain

Rationale: Clustering involves organizing patient assessment data into groupings with similar underlying causes. The nurse looks for cues among the data that support the diagnosis of a problem.
One patient may have several problems simultaneously, requiring the nurse to understand the potential relatedness of signs and symptoms from various body systems. The nurse combines an understanding of pathophysiology, normal structure and function, disease processes, and symptomatology to accurately cluster data.
A person who has not had a bowel movement may experience weight gain. Skin, nails, and hair are components of the integumentary system. The subjective feelings of tired and cold are related and prolonged menstruation, as part of the reproductive system, is in a group by itself.

80
Q

The nurse in an outpatient clinic obtains a blood pressure of 190/88 mm Hg on a patient diagnosed with high blood pressure. The patient tells the nurse, “My blood pressure medicine is really expensive. Do you think I really need it?” The nurse assumes the
patient is not taking the medication based on the blood pressure result and the patient’s statement and chooses lack of knowledge as a diagnostic label. The nurse identifies the action taken is an example of what concept of Nursing diagnosis formation?

a. Clustering unrelated data in the diagnostic statement
b. Selecting erroneous data for use in the diagnostic statement
c. Using medical diagnoses in the diagnostic statement
d. Identifying multiple problems within one diagnostic statement

A

a. Clustering unrelated data in the diagnostic statement

Rationale: A variety of errors in identification, statement structure, and statement content may occur when formulating Nursing diagnoses.
These include clustering unrelated data, accepting erroneous data, missing the true underlying etiology of a problem, using medical diagnoses as related factors in a NANDA-I Nursing diagnostic statement, and identifying multiple Nursing diagnosis labels in one
NANDA-I Nursing diagnostic statement.
Clustering unrelated data most often occurs when the nurse has not completed a thorough review of the patient’s assessment information or is missing important data. The nurse assumes the patient is not taking the blood pressure medication because of the cost and chooses the diagnosis of noncompliance. The nurse fails to ask the patient if the medication is being taken as ordered.
Errors in data collection (e.g., omitting key information) or an incomplete understanding or knowledge of assessment techniques or a patient’s condition may lead to the inclusion of erroneous data in a Nursing diagnostic statement. When writing Nursing diagnoses, the nurse should avoid inclusion of more than one label in the statement. Regardless of the type of Nursing diagnosis being written, only one label should be used in each statement. The nurse does not commit this error here. “Lack of knowledge” is not a medical diagnosis.

81
Q

The nurse is developing a plan of care for a patient with gastritis and inflammation of the intestines. The patient is complaining of severe abdominal discomfort and nausea. The patient also reports having restless leg syndrome and an inability to urinate. What should the nurse write as a problem statement for the Nursing diagnosis?

a. Gastritis related to inflammation.
b. Alterations in comfort and ability to void.
c. Abdominal pain and nausea related to inflammation.
d. Alteration in comfort related to restless leg syndrome and inflammation.

A

c. Abdominal pain and nausea related to inflammation.

Rationale: One patient may have several problems simultaneously, requiring the nurse to understand the potential relatedness of signs and symptoms from various body systems. The nurse combines an understanding of pathophysiology, normal structure and function, disease processes, and symptomatology to accurately cluster data. Abdominal pain, nausea, and inflammation (of the intestines) are clustered together.

82
Q

The nursing student submits a care plan to the nursing instructor for a review prior to implementing the nursing interventions. The instructor identifies which nursing diagnostic statement is written incorrectly?

a. Difficulty coping related to inadequate support systems as evidenced by patient’s
verbalization, “I don’t have any friends or family in town. I just moved here a week ago.”
b. Activity intolerance related to immobility as manifested by shortness of breath and patient’s verbalization of fatigue.
c. Impaired sleep and lack of knowledge related to stress as evidenced by patient report of difficulty sleeping and lack of energy.
d. Impaired self-feeding related to upper extremity weakness as manifested by inability to get food onto spoon.

A

c. Impaired sleep and lack of knowledge related to stress as evidenced by patient report of difficulty sleeping and lack of energy.

Rationale: To correctly formulate a nursing diagnostic statement, the student needs to cluster related data and choose one diagnosis per statement. In the incorrect example, two nursing diagnoses were combined in one statement.

83
Q

When creating a Nursing diagnosis, the nurse knows the related factor is based on what premise?

a. It should be based on the medical diagnosis.
b. It is unrelated to the pathophysiology causing the problem.
c. It is the underlying etiology of the patient’s situation.
d. It does not reflect the nurse’s understanding of pathophysiology.

A

c. It is the underlying etiology of the patient’s situation.

Rationale: The underlying etiology, or cause of a patient’s concern or situation, rather than a medical diagnosis, should be used as a related factor when writing a Nursing diagnosis. By doing so, the nurse articulates an understanding of the pathophysiology or situation with which the patient is faced.

84
Q

The nurse is caring for a complex patient needing physical and emotional support. As the primary caregiver, the nurse has which responsibility?

a. The nurse is ultimately responsible for assessment of patient needs and progress.
b. The nurse delegates to people who know what they are doing and operate independently.
c. The nurse provides total care to the patient after getting direction from other disciplines.
d. The nurse understands that the patient is ultimately responsible for failure or success.

A

a. The nurse is ultimately responsible for assessment of patient needs and progress.

Rationale: Even though collaboration and delegation may occur, the nurse is ultimately responsible for the continued assessment of patient needs and progress. As delegator, the nurse must supervise other disciplines to make sure that the patient needs are being met.
Detection of additional problems or lack of progress with the patient should prompt the nurse to reconsider the nursing process steps.

85
Q

The nurse has identified several problems for a patient scheduled for a bone marrow transplant. When formulating the Nursing diagnosis, the nurse includes which key concept?

a. The nurse realizes that changes in patient condition do not have to change diagnoses.
b. The nurse uses a language that is difficult to interpret by legislators.
c. The nurse can communicate with other nurses but not other disciplines.
d. The nurse facilitates communication of patient needs and promotes accountability.

A

d. The nurse facilitates communication of patient needs and promotes accountability.

Rationale: The use of Nursing diagnosis labels facilitates clear communication of patient needs and promotes professional accountability and autonomy by defining and describing the independent area of nursing practice.
Nursing diagnostic statements clearly communicate to legislators, consumers, and insurance providers the unique care nurses deliver and the specific nature of the health conditions they treat.
Use of a unified language classification system, or taxonomy, is an effective vehicle for communication among nurses and other health care professionals.

86
Q

The nurse is developing a plan of care for a patient who had a stroke. Assessment findings include weakness in right upper and lower extremities, numbness in face, slurred speech, difficulty with walking and balance, and headache. The nurse identifies which response would best represent the etiology of the patient’s gait and balance problems?

a. Lack of muscle motor movement
b. Decreased sensation to touch
c. Inability to speak clearly
d. Pain in back of head

A

a. Lack of muscle motor movement

Rationale: The related factor in an actual Nursing diagnosis needs to address the underlying etiology of the patient’s problem expressed by the
Nursing diagnostic label rather than listing data that are defining characteristics. The decreased sensation to touch, inability to speak clearly, and pain in the back of the head are only reiterations of the defining characteristics (numbness in face, slurred speech, and headache).

87
Q

The nurse is caring for a patient admitted to the intensive care unit with malnutrition. The patient is unable to walk and has developed a pressure ulcer from lying in bed constantly without changing positions. The family believes that the patient is
depressed and that is why getting out of bed has stopped. When planning this patient’s care, the nurse will include which key concept?

a. Develop multiple Nursing diagnoses.
b. Develop only one Nursing diagnosis to aid in focusing.
c. Focus on the physical issues facing this patient.
d. Deal primarily with the patient’s psychological needs.

A

a. Develop multiple Nursing diagnoses.

Rationale: Analysis of patient assessment data may yield several clusters of related data or cues. It is common to apply several Nursing diagnostic statements to one patient. This is especially true for acutely ill patients with multiple problems related to complex physical or psychological needs.

88
Q

The nurse is creating a care plan for a patient admitted with severe bone pain related to an infected leg wound. Which diagnosis written on the plan indicates an understanding of the components of a Nursing diagnosis? (Select all that apply.)

a. Acute pain
b. Risk for impaired walking
c. Ineffective bone tissue perfusion
d. Osteomyelitis
e. Infection

A

a. Acute pain
b. Risk for impaired walking
c. Ineffective bone tissue perfusion

Rationale: Whereas medical diagnoses identify and label medical (physical and psychological) illnesses, Nursing diagnoses are much broader in focus.
Nursing diagnoses consider a patient’s response to medical diagnoses and life situations in addition to making clinical judgments based on a patient’s actual medical diagnoses and conditions.
Pain, potential inability to ambulate, and decreased blood flow to the bone are a patient’s response to the medical condition of osteomyelitis.
Medical diagnoses identify the specific physical or psychological condition. Osteomyelitis and infection are medical diagnoses defined as inflammation and an infection of the bone usually caused by bacteria.

89
Q

The nurse is caring for a patient admitted to the psychiatric unit as a result of an overdose of cocaine. Which Nursing diagnosis indicates an understanding of a Nursing diagnostic statement? (Select all that apply.)

a. Impaired breathing related to drug effect on the respiratory center
b. Risk for injury related to hallucinations
c. Insomnia
d. Impaired socialization related to excessive stimulation of nervous system as evidenced by unintelligible speech.
e. Powerlessness

A

b. Risk for injury related to hallucination
d. Impaired socialization related to excessive stimulation of nervous system as evidenced by unintelligible speech.

Rationale: Each type of Nursing diagnostic statement contains sections or parts. Actual Nursing diagnostic statements are written with three parts: a diagnosis label, related factors, and defining characteristics.
Risk Nursing diagnoses have two segments: a diagnosis label and risk factors.
Health-promotion Nursing diagnoses are written with only two sections: the diagnosis label and defining characteristics.
The first statement needs defining characteristics. Insomnia is a medical diagnosis. The last statement needs etiology and manifestations

90
Q

A patient is admitted to the Emergency Department after experiencing severe chest pain and difficulty in taking deep breaths. The patient anxiously tells the nurse, “My father died suddenly of a heart attack at the age of 52. I’m so scared.” Which Nursing diagnoses are appropriate for this situation? (Select all that apply.)

a. Acute pain
b. Fear
c. Risk for aspiration
d. Risk for infection
e. Impaired role performance

A

a. Acute pain
b. Fear

Rationale: One patient may have several problems simultaneously, requiring the nurse to understand the potential relatedness of signs and symptoms from various body systems.
The nurse combines an understanding of pathophysiology, normal structure and function, disease processes, and symptomatology to accurately cluster data. The patient is reporting severe chest pain with an inability to take deep breaths.
The Nursing diagnostic label of acute pain is appropriate. Being scared is a defining characteristic of the Nursing diagnosis of fear. The patient is not at risk for aspiration or infection based on the data presented.

91
Q

A group of nursing students is discussing the importance of accurately selecting Nursing diagnoses. Which ideas offered in the students’ discussion are reasons for choosing the diagnoses carefully? (Select all that apply.)

a. Patient satisfaction
b. Positive patient outcomes
c. Quality patient care
d. Help develop standardized care plans
e. Determine appropriate interventions

A

a. Patient satisfaction
b. Positive patient outcomes
c. Quality patient care
e. Determine appropriate interventions

Rationale: Ultimately, nurses are accountable for formulating accurate Nursing diagnoses and intervening appropriately.
By collecting accurate and complete assessment data and articulating concise Nursing diagnoses for each patient, the professional nurse has a significant impact on patient care outcomes, the quality of patient care, and patient satisfaction.
By identifying and writing clear Nursing diagnostic statements, the nurse enables accurate development of individualized patient plans of care. Nursing diagnoses and patient outcomes, which are established during the planning step, help the nurse to determine appropriate interventions for patient care.

92
Q

The nurse has requested an order to place a patient on suicide watch. Which data noted in the health assessment led the nurse to this conclusion? (Select all that apply.)

a. Threats of killing oneself
b. Chronic pain
c. History of prior suicide attempt
d. Loneliness
e. Stable heart rhythm

A

a. Threats of killing oneself
b. Chronic pain
c. History of prior suicide attempt
d. Loneliness

Rationale: Risk factors may be environmental, physical, psychological, or situational concerns. The nurse is concerned that the patient may be at risk for suicide. Verbal statements by the patient, physical illness such as chronic pain, prior attempts to commit suicide and a lack of social interaction are potential causes for the act of suicide. A stable heart rhythm would not be a safety concern.

93
Q

The nurse is caring for a patient who has undergone abdominal surgery. The patient stated prior to surgery that “I don’t think I’ll be able to handle this if I get a colostomy. I wouldn’t know how to manage it.” The patient is complaining of severe surgical pain and
has an order for morphine sulfate. The nurse is correct when addressing which Nursing diagnosis first?

a. Pain
b. Alteration in body image
c. Knowledge deficit
d. Risk for falls

A

a. Pain

Rationale: Using Maslow’s hierarchy of needs helps to organize the most-urgent to less-urgent needs. This framework organizes patient data according to basic human needs common to all individuals.
Maslow’s theory suggests that basic needs, such as physiologic needs, must be met before higher needs, such as self-esteem.
The nurse also realizes that an actual problem takes priority over a potential problem. By using the nursing process appropriately, the nurse correctly chooses the actual, physiological problem first: pain.
Once the patient has the morphine, the risk for falls becomes a higher priority than knowledge deficit or alteration in body image because the morphine might confuse the patient, cause dizziness or faintness, and lead to a fall.

94
Q

Setting priorities among identified Nursing diagnoses is the first step in the planning process. The nurse knows this prioritization includes which action?

a. Monitoring patient responses
b. Carrying out the health care provider’s plan of care
c. Providing all interventions
d. Collaborating with other disciplines

A

a. Monitoring patient responses

Rationale: Setting priorities among identified Nursing diagnoses is the first step in the planning process. The nurse is responsible for monitoring patient responses, making decisions culminating in a plan of care, and implementing interventions, including interdisciplinary collaboration and referral, as needed. The nurse is significantly accountable for achieving the desired outcomes.

95
Q

Which assessment made by the nurse should be addressed first?

a. Reddened area to coccyx
b. Decreased urinary output
c. Shortness of breath
d. Drainage from surgical incision

A

c. Shortness of breath

Rationale: It is essential that the nurse identify life-threatening concerns and patient situations that need to be addressed most quickly.
The ABCs—airway, breathing, and circulation—are a valuable tool for directing the nurse’s thought process. Depending on the severity of a problem, the steps of the nursing process may be performed in a matter of seconds.
For instance, if a patient is in respiratory arrest, the most critical goal is for the patient to begin breathing. The reddened coccyx, decreased urinary output, and surgical incision drainage is not immediately life-threatening.

96
Q

Which patient issue should the nurse address first?

a. Pain
b. Hunger
c. Decreased self-esteem
d. Absence of pulse

A

d. Absence of pulse

Rationale: It is essential that the nurse identify life-threatening concerns and patient situations that need to be addressed most quickly.
The ABCs—airway, breathing, and circulation—are a valuable tool for directing the nurse’s thought process. Depending on the severity of a problem, the steps of the nursing process may be performed in a matter of seconds.
In this situation, the patient needs CPR immediately due to the absence of a pulse. Pain, hunger, and decreased self-esteem are not life-threatening issues. Although the nurse must address them, pulselessness is the priority.

97
Q

The nurse demonstrates a thorough understanding of the planning phase of the nursing process when making which statement?

a. “Patients should be included in the planning process.”
b. “Patient families should not interfere in the planning process.”
c. “The planning process should focus on short-term goals only.”
d. “Planning is the first phase of the nursing process.”

A

a. “Patients should be included in the planning process.”

Rationale: Planning is the third step of the nursing process. During the planning phase, the professional nurse prioritizes the patient’s Nursing diagnoses, determines short- and long-term goals, identifies outcome indicators, and lists nursing interventions for patient-centered care.
Patients should be included in the planning process. Involving patients in planning their care helps them to: (1) be aware of identified needs, (2) accept realistic and measurable goals, and (3) embrace interventions to best achieve the mutually agreed-on goals.
Inclusion of patients in the planning process tends to improve goal attainment and patient cooperation with interventions.
Depending on the patient’s condition or circumstances, it may be advantageous to include members of the patient’s support system (i.e., family, friends, and caregivers) in the planning phase.

98
Q

The nurse recognizes that patient goals include which characteristic?

a. They are considered short-term if achieved within a month of identification.
b. They always have established time parameters, such as “long-term” or “short-term.”
c. They are mutually acceptable to the nurse, patient, and family.
d. They can be vague to facilitate flexibility when evaluating achievement.

A

c. They are mutually acceptable to the nurse, patient, and family.

Rationale: Goals are broad statements of purpose that describe the aim of nursing care. Goals represent short- or long-term objectives that are determined during the planning step.
Some sources establish time parameters for short- and long-term goals, whereas others do not.
According to Carpenito, goals that are achievable in less than a week are short-term goals, and goals that take weeks or months to achieve are long-term goals. Useful and effective goals have certain characteristics. They are mutually acceptable to the nurse,
patient, and family. They are appropriate in terms of nursing and medical diagnoses and therapy.
The goals are realistic in terms of the patient’s capabilities, time, energy, and resources, and they are specific enough to be understood clearly by the patient and other nurses. They can be measured to facilitate evaluation.

99
Q

When developing the nursing care plan, the nurse includes which concept when creating goals?

a. Develops the goals with the patient and possibly the family.
b. Creates the goals that the nurse wants the patient to achieve.
c. Includes the actions that are needed to accomplish the goal.
d. Focus on goals that are aggressive to ensure success.

A

a. Develops the goals with the patient and possibly the family.

Rationale: The nurse creates goals with the patient and possibly with the family by discussing the patient’s current condition, the condition to which the patient wants to progress, and the actions the patient and nurse undertake to accomplish the goal.
If the goals are simply what the nurse wants the patient may have other goals in mind to which he or she gives more attention and effort.
The goal does not include the interventions or actions needed; however they must be discussed so the patient understands the care he or she is to receive and what part the patient plays in achieving the goals. The nurse works with the patient to develop a plan of care that is appropriately challenging and promotes patient success in attaining goals.

100
Q

Which statement by the nurse is correct regarding diversity considerations?

a. The male gender may struggle less with health care terminology.
b. High numbers of minority populations do not understand health teachings.
c. Older adults understand health teaching easily because of life experience.
d. Disabilities have no impact on the development of patient care goals.

A

b. High numbers of minority populations do not understand health teachings.

Rationale: Minority patients globally and those with low English proficiency in the United States are especially vulnerable to low health literacy.
Patients of both genders, including those who are well educated and highly literate but have limited health care experience, may struggle with the complexity of health care terminology and procedures.
Older adults have also been identified as a group experiencing low health literacy. Before implementing teaching strategies to support goal attainment, the nurse must explore a patient’s disabilities and the effects they may have on achieving specific goals. Successful accommodation of a patient’s disabilities should yield attainable goals that lead to positive outcomes.

101
Q

The nurse recognizes which is a correctly written example of a short-term goal?

a. The patient will lose 50 lb in 1 year.
b. Patient will ambulate 1 mile without shortness of breath.
c. Patient will be able to change the colostomy bag in 6 weeks.
d. The patient will eat 75% of all meals for the next three days.

A

d. The patient will eat 75% of all meals for the next three days.

Rationale: According to Carpenito, goals that are achievable in less than a week are short-term goals, and goals that take weeks or months to achieve are long-term goals.
Eating 75% of all meals for the next three days is an example of a short-term goal. A goal set for 6 weeks or a year in the future is a long-term goal. The ambulation goal has no time determination; therefore it can’t be labeled as either short- or long-term and as such, is incorrectly written.

102
Q

The nurse identifies which goal is written correctly for the Nursing diagnosis of activity intolerance related to imbalance between oxygen supply and demand?

a. Patient will walk 1 mile without shortness of breath.
b. Patient will ambulate 100 feet with no shortness of breath on third day after treatment.
c. Patient will climb stairs without shortness of breath by day 2 of hospital stay.
d. Patient will tolerate activity.

A

b. Patient will ambulate 100 feet with no shortness of breath on third day after treatment.

Rationale: Useful and effective goals have certain characteristics. They are appropriate in terms of nursing and medical diagnoses and therapy.
The goals are realistic in terms of the patient’s capabilities, time, energy, and resources, and they are specific enough to be understood clearly by the patient and other nurses.
They can be measured to facilitate evaluation. In option A, there is no time frame to gauge expectations, so the diagnosis is not measurable. In option C, the number of stairs is not specified and so is not measurable. In option D, the type of activity is not mentioned, so it is not specific and there is no measurable criterion.

103
Q

The nurse recognizes which response as a barrier to achieving goals?

a. The effects of pain and/or clinical depression
b. Patient involvement in setting patient goals
c. Family involvement in setting patient goals
d. Realistic expectations of the patient’s capabilities

A

a. The effects of pain and/or clinical depression

Rationale: Pain and depression both can lead the patient away from cooperation and motivation in working towards goal achievement.
The nurse must address either problem in order to have the best chance of success for the patient in meeting goals.
Patient and family involvement help create “buy-in” and cooperation. Realistic expectations lead to reasonable and achievable goals.

104
Q

The nurse is caring for a patient who has had abdominal surgery and has developed a slight temperature. The nurse identifies which
statement to be a patient-centered goal?

a. The patient’s temperature will return to normal within 24 hours.
b. The nurse will medicate the patient for elevated temperature every 4 hours as
needed.
c. Skin integrity will be maintained until the patient is ambulatory.
d. The patient will ambulate 10 feet by postoperative day 2.

A

d. The patient will ambulate 10 feet by postoperative day 2.

Rationale: Patient-centered goals are written specifically for the patient. The goal should specify the activity the patient is to exhibit or demonstrate to indicate goal attainment.
These goals are written to reflect patient, not nursing, activities. Instead of focusing on the patient, the incorrect answers focus on the patient’s temperature, the nurse medicating the patient, and the patient’s skin integrity.
Only option D focuses on the patient.

105
Q

The nurse knows which response to be an example of a measurable goal?

a. “The patient will be able to lift 10 lb. by the end of week one.”
b. “The patient will be able to lift weights by the end of the week.”
c. “The patient will be able to lift his normal weight amount.”
d. “The patient will be able to lift an acceptable amount of weight by week one.”

A

a. “The patient will be able to lift 10 lb. by the end of week one.”

Rationale: Measurable goals are specific, with numeric parameters or other concrete methods of judging whether the goal was met.
When writing a goal statement with a patient, the nurse needs to clearly identify how achievement of the goal will be evaluated.
When terms such as acceptable or normal are used in a goal statement, goal attainment is difficult to judge because they are not measurable terms, unless they refer to laboratory values or diagnostic test findings.
The amount of weight a patient will lift at the
end of the week is not specified. “Normal” and “acceptable” weight have not been defined.

106
Q

The nurse is formulating the patient’s care plan. In determining when to evaluate the patient’s progress, the nurse is aware that evaluations should be carried out within which parameters?

a. They must be done at the end of every shift.
b. They should be done at least every 24 hours.
c. They depend on intervention and patient condition.
d. They are always done at time of discharge.

A

c. They depend on intervention and patient condition.

Rationale: In most cases, goal statements need to include a time for evaluation. The time depends on the intervention and the patient’s condition. Some goals may need to be evaluated daily or weekly, and others may be evaluated monthly.
The health care setting affects the time of evaluation. If the goal is set during hospitalization, the goal may need to be evaluated within days, whereas a goal set for home care may be evaluated weekly or monthly.
At the time of evaluation, the goal is assessed for goal attainment, and new goals are set or a new evaluation date for the same goal may be chosen if the goal is still applicable for the patient care plan.

107
Q

The nurse knows that standardized care plans may be available and are utilized under which circumstance?

a. They need to be individualized for each patient.
b. They are implemented without adjustment.
c. They remove the need for nurse involvement.
d. They do not require the use of Nursing diagnoses.

A

a. They need to be individualized for each patient.

Rationale: There are multiple formats in which to develop individualized care plans for patients, families, and communities. Each health care agency has its own form, including electronic formats, to facilitate the documentation of patient goals and individualized
patient-centered plans of care.
All formats contain areas in which the nurse identifies key assessment data, Nursing diagnostic statements, goals, interventions for care, and evaluation of outcomes. In many agencies and specialty units, standardized care plans that must be individualized for each patient are available to guide nurses in the planning process.

108
Q

The nurse recognizes which term identifies nursing interventions that originate from the health care provider orders?

a. Dependent
b. Independent
c. Collaborative
d. Nursing interventions classifications

A

a. Dependent

Rationale: Interventions originating from a provider’s order are dependent nursing interventions. Independent nursing interventions are originated by the nurse based on expertise in meeting patient needs or preventing complications.
Interventions that include collaboration with other providers, such as physical therapy, are collaborative interventions. Nursing Interventions Classification (NIC) is a research-based, standardized collection of interventions and associated activities.

109
Q

The nurse identifies medication administration to be what type of nursing intervention?

a. Independent
b. Dependent
c. Collaborative
d. Interdisciplinary

A

b. Dependent

Rationale: Interventions originating from a provider’s order are dependent nursing interventions.
Independent nursing interventions are originated by the nurse based on expertise in meeting patient needs or preventing complications.
Interventions that include collaboration with other providers, such as physical therapy, are collaborative interventions.
Collaborative interventions require cooperation among a few or many members of the interdisciplinary health care team.

110
Q

The nurse recognizes which action to be a dependent nursing intervention?

a. Utilizing heel protectors
b. Preadmission teaching
c. Medication reconciliation
d. Oxygen administration via mask

A

d. Oxygen administration via mask

Rationale: Dependent nursing interventions originate from health care provider orders. These interventions include orders for oxygen administration, dietary requirements, medications and diagnostic tests.
The nurse incorporates these orders into the patient’s overall care enact independent interventions has expanded in recent years, allowing nurses to initiate care that they recognize as essential in meeting patient needs or preventing complications.
Utilizing heel protectors for patients susceptible to skin breakdown and initiating preventive measures (e.g., activity regimens, consultations with social workers, preadmission teaching) are often independent, nurse-initiated interventions.
Collaborative interventions require cooperation among several health care professionals and unlicensed assistive personnel (UAP).
Collaborative interventions include activities such as physical therapy, home health care, personal care, spiritual counseling, medication reconciliation, and palliative or hospice care.

111
Q

The nurse recognizes that physical therapy, speech therapy, home health care, and personal care are examples of which type of interventions?

a. Collaborative interventions
b. Dependent nursing interventions
c. Independent nursing interventions
d. Assessment interventions

A

a. Collaborative interventions

Rationale: Interventions that include collaboration with other providers, such as physical therapy, are collaborative interventions.
Interventions originating from a provider’s order are dependent nursing interventions. Independent nursing interventions are originated by the nurse based on expertise in meeting patient needs or preventing complications. An assessment is done to gather
data.

112
Q

The nurse understands that discharge planning begins at what point in the patient’s hospitalization?

a. The day before discharge
b. Upon admission
c. Prior to admission
d. Day of discharge

A

b. Upon admission

Rationale: Discharge planning plays an important role in the success of a patient’s transition to the home setting after hospitalization.
Because most patients are in the hospital for only a short time, nurses must begin discharge planning on admission and continue until a patient is dismissed.

113
Q

The nurse identifies which statement to be accurate regarding discharge planning?

a. “It may decrease the incidence of patients who need to return to the hospital.”
b. “It increases complications and readmissions in most cases.”
c. “It adapts to the situation as the patient’s conditions changes.”
d. “It should begin as soon as the patient is discharged home.”

A

a. “It may decrease the incidence of patients who need to return to the hospital.”

Rationale: Research shows that comprehensive discharge planning reduces complications and readmissions.
Home care planning adapts to the situation as the patient’s condition improves or deteriorates as a result of advancing disease.
Because most patients are in the hospital for only a short time, nurses must begin discharge planning on admission and continue until a patient is dismissed.

114
Q

Since in the planning phase, the significance of developing organized plans of care for patients is important, the nurse must take seriously which of these responsibilities? (Select all that apply.)

a. Prioritizing patient needs
b. Developing mutually agreed-on goals
c. Determining outcome criteria
d. Identifying interventions
e. Implementation of the patient’s plan of care

A

a. Prioritizing patient needs
b. Developing mutually agreed-on goals
c. Determining outcome criteria
d. Identifying interventions

Rationale: The significance of developing organized plans of care for patients cannot be stressed enough. The nurse must take seriously the responsibility of prioritizing patient needs, developing mutually agreed-on goals, determining outcome criteria, and identifying
interventions that can help patients to achieve positive outcomes.
After these actions are completed in the planning phase of the nursing process, it is time for implementation of the patient’s plan of care (implementation phase).

115
Q

The nurse is formulating a plan of care for a patient. In this phase of the nursing process, the nurse should complete which actions? (Select all that apply.)

a. Prioritize Nursing diagnoses.
b. Determine short- and long-term goals.
c. Identify outcome indicators.
d. List nursing interventions.
e. Gather assessment data.

A

a. Prioritize Nursing diagnoses.
b. Determine short- and long-term goals.
c. Identify outcome indicators.
d. List nursing interventions.

Rationale: Planning is the third step of the nursing process. During the planning phase, the professional nurse prioritizes the patient’s Nursing diagnoses, determines short- and long-term goals, identifies outcome indicators, and lists nursing interventions for patient-centered care.
Each of these actions requires careful consideration of assessment data (collected earlier) and a thorough understanding of the relationship among Nursing diagnoses, goals, and evidence-based interventions.

116
Q

The nurse recognizes that by involving the patient in planning care, which patient results occur? (Select all that apply.)

a. Being aware of identified needs
b. Accepting that not all goals are measurable
c. Embracing mutually agreed-on goals
d. Feeling a sense of empowerment
e. Overcoming unrealistic goals

A

a. Being aware of identified needs
c. Embracing mutually agreed-on goals
d. Feeling a sense of empowerment

Rationale: Patients should be included in the planning process. Involving patients in planning their care helps them to: (1) be aware of identified needs, (2) accept realistic and measurable goals, and (3) embrace interventions to best achieve the mutually agreed-on
goals. Inclusion of patients in the planning process tends to improve goal attainment and patient cooperation with interventions.
By accepting guidance and input from patients during the planning process, the nurse provides them with a greater sense of empowerment and control.

117
Q

The nurse recognizes measurable goal to have which characteristics? (Select all that apply.)

a. Specific
b. Concrete
c. Vague
d. Easy to judge
e. Nonspecific

A

a. Specific
b. Concrete
d. Easy to judge

Rationale: Measurable goals are specific, with numeric parameters or other concrete methods of judging whether the goal was met.
When writing a goal statement with a patient, the nurse needs to clearly identify how achievement of the goal will be evaluated.
When terms such as acceptable or normal are used in a goal statement, goal attainment is difficult to judge because they are not measurable terms, unless they refer to laboratory values or diagnostic test findings.

118
Q

The nurse identifies which action as a direct-care intervention?

a. Administration of an injection
b. Making the change-of-shift report
c. Collaborating with members of the health care team
d. Ensuring availability of needed equipment

A

a. Administration of an injection

Rationale: Direct care refers to interventions that are carried out by having personal contact with patients. For example, direct-care interventions include cleaning an incision, administering an injection, ambulating with a patient, and completing patient teaching at
the bedside.
Indirect care includes nursing interventions that are performed to benefit patients but do not involve face-to-face contact with patients.
Examples of indirect care include making the change-of-shift report, communicating and collaborating with
members of the interdisciplinary health care team, and ensuring availability of needed equipment.

119
Q

The nurse manager is creating the patient assignment for today. She has five registered nurses (RNs), two licensed practical nurses (LPNs), and five nurse technicians (NAs) scheduled. When making the assignment, the nurse manager needs to remember which fact of delegation?

a. RNs are responsible for all care delegated to unlicensed nursing personnel.
b. Delegation is considered direct intervention for patient care.
c. LPNs operate independently and may delegate patient care.
d. Nursing practice is clearly delineated and is standard across the country.

A

a. RNs are responsible for all care delegated to unlicensed nursing personnel.

Rationale: Delegation is the transfer of responsibility for performing a task to another person while the nurse who delegated the task remains accountable. Delegation is an indirect intervention based on assessment findings and established care priorities. Nurses must be familiar with the nurse practice act in their practice jurisdiction to ensure legal delegation. The nursing process cannot be delegated.
In most jurisdictions, LPNs function in a dependent role and may not delegate.

120
Q

The nurse is preparing to administer medications to a patient. When the patient reports new shortness of breath, which action by the nurse is most appropriate?

a. Provide the patient with oxygen since it does not require a provider order.
b. Complete at least three checks to ensure that the proper medication is given.
c. Check the provider orders for all forms of prescription medications.
d. Document that the 6 rights of medication administration were followed.

A

c. Check the provider orders for all forms of prescription medications.

Rationale: All forms of prescription medication (i.e., oral, topical, and parenteral) require an order before administration, as does providing oxygen to a patient. The nurse would check for an as needed order for oxygen.
Nurses must complete three checks, follow the sixrights of medication administration, and document appropriately when administering medications, but those actions are not the priority due to the change in the patient’s condition. The nurse must first address the patient’s shortness of breath.

121
Q

After completing a patient’s initial assessment and developing a plan of care, what action by the nurse is most appropriate?

a. Continuously reassess the patient.
b. Restrict changes to the care interventions.
c. Reassess the patient at the start of each shift.
d. Evaluate patient goal attainment at intervals.

A

a. Continuously reassess the patient.

Rationale: After the nurse completes a patient’s initial assessment and develops a plan of care, continual reassessment of the patient detects noticeable changes in the patient’s condition, requiring adjustments to interventions outlined in the plan of care. The need for continual patient reassessment underscores the dynamic nature of the nursing process and is crucial to providing essential care.

122
Q

The male nurse is caring for a female patient who needs a complete bed bath. The patient requests that a female nurse bathe her.
The male nurse recognizes this request as an example of what type of diversity?

a. Gender diversity involving generational norms.
b. Life span diversity
c. Disability diversity
d. Morphology diversity

A

a. Gender diversity involving generational norms.

Rationale: The nurse must perform the procedures competently and safely, taking into consideration any special needs of the patient.
Gender diversity occurs with the identification of gender roles that may affect care delivery. Some patients may prefer care from nurses of the same gender.
This preference may stem from generational norms, personal comfort, or cultural considerations. With life span diversity, interventions must always be age or developmental level appropriate.
Disability diversity requires that interventions be individualized for each patient and adapted for any limitations.
The nurse must ensure safe practice in relation to patient body size (morphology diversity) and should seek additional support or equipment when necessary.

123
Q

The nurse is providing care for a patient of the Jehovah’s Witness faith. Based on the nurse’s knowledge of the patient’s religious beliefs, the nurse would question which order?

a. Obtain vital signs every shift.
b. Regular diet as tolerated.
c. Activity as tolerated.
d. Infuse 1 unit packed red blood cells.

A

d. Infuse 1 unit packed red blood cells.

Rationale: Some interventions may be declined because of religious affiliation (e.g., blood transfusion for a Jehovah’s Witness, pork-based insulin for a Muslim patient).

124
Q

The nurse is caring for a patient who is blind. When reviewing the care plan, the nurse would modify which goal?

a. The patient will report any drainage from the wound with a foul odor to the primary care provider after discharge.
b. The patient will agree to report pain promptly while hospitalized.
c. The patient will obtain no injuries while in the hospital.
d. The patient will report any purulent wound drainage to the primary care provider after discharge

A

d. The patient will report any purulent wound drainage to the primary care provider after discharge

Rationale: Interventions must be individualized for each patient and adapted for any limitations (e.g., amputation, learning disability, blindness, deafness). The patient would be able to detect a foul odor, report pain, and remain injury-free, but would not be able to tell if drainage is purulent.

125
Q

The registered nurse is providing an independent nursing intervention when completing which action?

a. Administering oral medications
b. Administering oxygen
c. Providing emotional support
d. Administering intravenous medication

A

c. Providing emotional support

Rationale: Independent nursing interventions are tasks within the nursing scope of practice that the nurse may undertake without a physician or PCP order. Repositioning a patient in bed, performing oral hygiene, and providing emotional support through active listening are examples of independent nursing interventions.
Dependent nursing interventions are tasks the nurse undertakes that are within the
nursing scope of practice but require the order of a primary care provider to be implemented. Administering patient medications or administering oxygen to a patient are examples of common dependent nursing interventions that require clinical judgment before implementation.
These interventions are based on a collaborative effort of the nurse and the physician to provide care to patients.

126
Q

The nurse recognizes which topic is appropriate teaching content for the patient who is returning from surgery?

a. Signs and symptoms of infection
b. Use of patient-controlled analgesia
c. Activity limitations upon discharge
d. Physical therapy

A

b. Use of patient-controlled analgesia

Rationale: Readiness to learn is an important consideration. For example, when a patient returns from surgery, it is essential that some information be reviewed (e.g., how to use the patient-controlled analgesia pump and incentive spirometer) but completing all discharge teaching at this time would not be effective.
At other times, teaching is more formalized, such as discharge teaching, signs of infection, and physical therapy.

127
Q

The nurse is learning to identify readiness to learn in patients. Which patient would the nurse identify correctly as ready to learn?

a. The patient requesting pain medication for treatment of severe discomfort
b. The patient reporting nausea and vomiting
c. The patient who was just told the diagnosis of cancer of the pancreas
d. The patient who was recently diagnosed with diabetes mellitus and is scheduled to be discharged in 2 days

A

d. The patient who was recently diagnosed with diabetes mellitus and is scheduled to be discharged in 2 days

Rationale: Choosing opportunities when the patient’s condition and environment are most conducive to learning is recommended when attempting to teach patients. Patients who are in pain, are nauseated, or who have been given recent traumatic diagnoses are not psychologically able to retain information.

128
Q

The nurse asks the patient for permission to involve the patient’s family members in the teaching plan for the patient. Which response is the best rationale to support this involvement?

a. Involving the family empowers the patients and their support system.
b. Teaching family members decreases the number of questions they may ask.
c. Educated family members ensure the patient will comply with the treatment plan.
d. The family members may be interested in the information.

A

a. Involving the family empowers the patients and their support system.

Rationale: With the patient’s permission, the nurse should share instructions with the people who may assist with care. Nurses empower patients and their support systems through effective teaching.
When nurses provide patients and their families with opportunities to ask questions and comprehend health care information, they become an integral part of the health care process.
The family members may ask fewer questions but that is not a reason to involve them. Nothing will ensure patient compliance other that the patient deciding to do so. Family members may be interested in the information, but that is not the main reason to include them.

129
Q

The nurse identifies change-of-shift report, collaboration with other health care members, and ensuring availability of needed equipment are examples of which term?

a. Indirect care
b. Direct care
c. Referrals
d. Delegation

A

a. Indirect care

Rationale: Indirect care includes nursing interventions that are performed to benefit patients but do not involve face-to-face contact with patients. Examples of indirect care include making the change-of-shift report, communicating and collaborating with members of
the interdisciplinary health care team, and ensuring availability of needed equipment.
Direct care refers to interventions that are carried out by having personal contact with patients. For example, direct-care interventions include cleaning an incision,
administering an injection, ambulating with a patient, and completing patient teaching at the bedside. Referrals in health care involve sending a patient to another member of the interdisciplinary health care team for a consultation or other services.
Delegation is the transfer of responsibility for performing a task to another person while the nurse who delegated the task remains accountable.

130
Q

The nurse correctly identifies which referral as an inappropriate nursing referral?

a. Music therapist
b. Community agencies
c. Adaptive care services
d. Dermatologist

A

d. Dermatologist

Rationale: A primary care provider (PCP) may refer a patient to a medical or surgical specialist for further assessment, testing, or treatment.
Nurses are often instrumental in initiating these types of referrals but do not complete the actual referral. Referral to a community agency is usually a collaborative action. Obtaining adaptive services and music therapy are independent nursing actions.

131
Q

When implementing research-based interventions, the nurse realizes which concept?

a. Implementing evidence-based care is unique to the nursing profession.
b. Evidence-based practice is based entirely in nursing research.
c. Evidence-based care is focused on practices and not outcomes.
d. Nurses must read recent literature and remain current in practice.

A

d. Nurses must read recent literature and remain current in practice.

Rationale: To implement research-based interventions, nurses must-read recent literature and remain current in practice. Implementation of evidence-based care is not unique to nursing; it involves interventions provided by all members of the interdisciplinary health care team.
The best methods for treating patients with a variety of signs and symptoms are researched by nurses with input from the research findings of other disciplines. Nursing care continues to evolve as nursing research provides new knowledge and recognizes best practices to improve patient care and outcomes. Evidence-based practice guidelines and updated information must be included in plans of care.

132
Q

When the nurse is supportive and works of behalf of patients, this role is identified by which term?

a. Advocate
b. Primary care provider
c. Collaborator
d. Delegator

A

a. Advocate

Rationale: Nurses advocate by supporting and working on behalf of patients or persons for whom they have concern. Nurses advocate for patients by coordinating care and supporting the changes necessary to improve conditions and outcomes.
Effective communication
and collaboration regarding patient care are essential for patient safety and positive patient outcomes. The change-of-shift reports are an example.
A PCP is usually a physician or advance practice nurse. Delegation is the transfer of responsibility for performing a task to another person while the nurse who delegated the task remains accountable.

133
Q

The nurse recognizes which task that cannot be delegated?

a. Obtaining vital signs
b. Assessing lung sounds
c. Bathing a patient
d. Ambulating a patient

A

b. Assessing lung sounds

Rationale: Delegation is the transfer of responsibility for performing a task to another person while the nurse who delegated the task remains accountable. Obtaining vital signs, bathing, and ambulating are all tasks associated with the assessment part of the nursing process.
The nursing process cannot be delegated.

134
Q

The nurse identifies repositioning a patient, providing hygiene, and active listening as examples of what concept?

a. Dependent interventions
b. Independent nursing interventions
c. Standing orders
d. Counseling

A

b. Independent nursing interventions

Rationale: Independent nursing interventions are tasks within the nurse’s scope of practice and do not require an order from a physician.
Dependent nursing interventions are tasks the nurse undertakes that are within the nursing scope of practice but require the order of a primary care provider to be implemented.
Some physician orders are received through a preapproved standardized order set known as standing orders. Counseling is the process through which individuals use professional guidance to address personal conflicts or emotional problems.

135
Q

The patient has an order for morphine sulfate 2 mg intravenously prn (as needed) every 2 hours. When the nurse administers this medication, which concept is being provided?

a. Independent nursing intervention
b. Dependent nursing intervention
c. Referral
d. Indirect care procedure

A

b. Dependent nursing intervention

Rationale: Dependent nursing interventions are tasks that require an order from a physician or primary care provider (PCP).
Independent nursing interventions are tasks within the nursing scope of practice that the nurse may undertake without a physician or PCP order.
Referrals in health care involve sending a patient to another member of the interdisciplinary health care team for a consultation or other services.
Indirect care includes nursing interventions that are performed to benefit patients but do not involve face-to-face contact with patients.

136
Q

The nurse understands which essential fact regarding documentation?

a. It should be completed accurately and in a timely manner.
b. It should not be computerized (EHR) because of disclosure risks.
c. It is not a legal document although they can be helpful in lawsuits.
d. It cannot be used in determining billing and reimbursement issues.

A

a. It should be completed accurately and in a timely manner.

Rationale: All documentation entries should be completed in a timely, accurate, and professional manner. Documentation most often is charted in the patient’s EHR and standardized flow sheets according to agency policy. Patient health records are legal documents.
Within the Health Insurance Portability and Accountability Act (HIPAA) guidelines, patient documentation is provided to insurance companies and others for billing and reimbursement.

137
Q

The nurse knows what fact to be the focus of evaluation, the final phase of the nursing process?

a. The focus is recording the care that was implemented.
b. The focus is medical and nursing goals for the welfare of the patient.
c. The focus is long-term goals only.
d. The focus is patient responses to interventions and outcomes.

A

d. The focus is patient responses to interventions and outcomes.

Rationale: Evaluation is the final step in the nursing process. Evaluation focuses on the patient and the patient’s response to nursing interventions and outcome attainment.
Evaluation is not a record of care that was implemented. Patient outcomes serve as the
criteria against which the success of a nursing intervention is judged.
During the evaluation phase, nurses use critical thinking to determine whether a patient’s short- and long-term goals were met and whether desired outcomes were achieved.
Monitoring whether the patient’s goals were attained is collaborative, involving the patient in the decision-making process.

138
Q

When the nurse realizes that the patient’s short-term goals have not been met, the nurse should carry out which task?

a. Revise or adapt the plan of care.
b. Assume that the patient did not want to achieve his goals.
c. Understand that a plan of care is almost never changed.
d. Reassess plans of care only after major patient-nurse interactions.

A

a. Revise or adapt the plan of care.

Rationale: When a patient goal is unmet or only partially met, the plan of care may need to be revised or adapted to support goal attainment.
There are many reasons why goals are not met, including changes in the patient condition, unrealistic goals, or inappropriate interventions that do not help meet the goal. It is common for plans of care to change to meet evolving needs.
Reassessment occurs with each patient-nurse interaction. As changes in a patient’s condition occur, the plan of care should be revised.

139
Q

The nurse identifies that the nursing process is an attempt to meet patient needs, including which concept?

a. Nursing process is linear in nature.
b. Nursing process is dynamic and cyclic.
c. Nursing process requires occasional care plan re-evaluation.
d. Nursing process does not allow care plan modification.

A

b. Nursing process is dynamic and cyclic.

Rationale: The nursing process is ongoing in an attempt to meet patient needs.
The nursing process is not linear in nature but is dynamic and cyclic, constantly adapting to a patient’s health status.
Care plan modifications may be necessitated due to deterioration or improvement of a patient’s condition. The Joint Commission requires patient care plans to be evaluated on a continual basis.

140
Q

The nurse recognizes which interventions to be prevention oriented? (Select all that apply.)

a. Immunization programs
b. Cleansing an incision
c. Cardiac risk factor modification
d. Placing infants prone when they sleep
e. Teaching patients to ask their providers to wash their hands

A

a. Immunization programs
b. Cleansing an incision
c. Cardiac risk factor modification
d. Placing infants prone when they sleep
e. Teaching patients to ask their providers to wash their hands

Rationale: Some interventions prevent illness or complications and promote healthy activities or lifestyles. Interventions such as patient education and immunization programs are prevention-oriented. Cleansing an incision is a nursing intervention that can help prevent infection. Educating a patient about risk-factor modification for cardiovascular disease may prevent a future myocardial infarction.
Placing infants on their backs to sleep may reduce the risk of sudden infant death syndrome. Patients should be instructed to ask their care providers to wash their hands if they have not observed them doing so.

141
Q

The nurse considers which skills to be invasive procedures? (Select all that apply.)

a. Administering oral medications
b. Starting an intravenous (IV) line
c. Repositioning the patient
d. Inserting a urinary catheter

A

b. Starting an intravenous (IV) line
d. Inserting a urinary catheter

Rationale: Many interventions focus on physical care that is performed when treating patients. These interventions may include invasive procedures, such as starting an intravenous line or inserting a catheter, or they may be noninvasive, such as administering oral medications and repositioning.

142
Q

The nurse understands that the five rights of delegation include which components? (Select all that apply.)

a. Right patient
b. Right time
c. Right person
d. Right supervision
e. Right task

A

c. Right person
d. Right supervision
e. Right task

Rationale: Delegation principles focus on the appropriate intervention (task) being performed under the correct circumstances, by the correct personnel, and with the correct direction and supervision. The right patient and the right time refer to components of the “6 Rights” of medication administration.

143
Q

Which set of assessment data is consistent for a patient with severe infection that could lead to system failure?

a. Blood pressure (BP) 92/52, pulse (P) 56 beats/min, respiratory rate (RR) 10 breaths/min, urine output 1200 mL in past 24 hours
b. BP 90/48, P 112 beats/min, RR 26 breaths/min, urine output 240 mL in past 24 hours
c. BP 112/64, P 98 beats/min, RR 18 breaths/min, urine output 2400 mL in past 24 hours
d. BP 152/90, P 52 beats/min, RR 12 breaths/min, urine output 4800 mL in past 24 hours

A

b. BP 90/48, P 112 beats/min, RR 26 breaths/min, urine output 240 mL in past 24 hours

Rationale: The patient with severe infection presents with low BP and compensating elevations in pulse to move lower volumes of blood more rapidly and respiration to increase access to oxygen.
Urine output decreases to counteract the decreased circulating blood volume and hypotension.
These vital signs are all too low: Blood pressure (BP) 92/52, pulse (P) 56 beats/min, respiratory rate
(RR) 10 breaths/min, urine output 1200 mL in past 24 hours.
The patient with severe infection does have a low BP, but the pulse and respiratory rate increase to compensate.
This data is all within normal limits: BP 112/64, P 98 beats/min, RR 18 breaths/min, urine output 2400 mL in past 24 hours.
This set of data reflects an elevated BP with a decrease in pulse and respiratory rates along with normal urine output: BP 152/90, P 52 beats/min, RR 12 breaths/min, urine output 4800 mL in past 24 hours.
None of these is a typical response to severe infection.

144
Q

The nurse is working on a plan of care with her patient which includes turning and positioning and adequate nutrition to help the patient maintain intact skin integrity. The nurse helps the patient to realize that this breaks the chain of infection by eliminating which element?

a. Host
b. Mode of transmission
c. Portal of entry
d. Reservoir

A

c. Portal of entry

Rationale: Broken or impaired skin creates a portal of entry for pathogens. By maintaining intact tissue, the patient and the nurse have broken the chain of infection by eliminating a portal of entry.
Host is incorrect because you are not eliminating the person or organism. Intact tissue does not eliminate the mode of transmission. Skin can still be used to transfer pathogens regardless of it being intact or broken. Intact skin does not eliminate the location for pathogens to live and grow.

145
Q

While reviewing the complete blood count (CBC) of a patient on her unit, the nurse notes elevated basophil and eosinophil readings. The nurse realizes that this is most indicative of which type of infection?

a. Bacterial
b. Fungal
c. Parasitic
d. Viral

A

c. Parasitic

Rationale: Parasitic infections are frequently indicated on a CBC by elevated basophil and eosinophil levels. Bacterial infections do not lead to elevated basophil and eosinophil levels but elevated B and T lymphocytes, neutrophils, and monocytes.
Fungal infections do not lead to elevated basophil and eosinophil levels. Viral infections create elevations in B and T lymphocytes, neutrophils, and monocytes.

146
Q

A nurse is teaching a group of businesspeople about disease transmission. The nurse knows that additional teaching is needed when one of the participants states which of the following?

a. “When traveling outside of the country, I need to be sure that I receive appropriate vaccinations.”
b. “Food and water supplies in foreign countries can contain microorganisms to which my body is not accustomed and has no resistance.”
c. “If I don’t feel sick, then I don’t have to worry about transmitted diseases.”
d. “I need to be sure to have good hygiene practices when traveling in crowded planes and trains.”

A

c. “If I don’t feel sick, then I don’t have to worry about transmitted diseases.”

Rationale: People can transmit pathogens even if they don’t currently feel ill. Some carriers never experience the full symptoms of a pathogen.
Travelers may need different vaccinations when traveling to countries outside their own because of variations in prevalent microorganisms.
Food and water supplies in foreign countries can contain microorganisms that will affect a body unaccustomed to their presence.
Adequate hygiene is essential when in crowded, public spaces like planes and other forms of public transportation.

147
Q

In order to provide an intervention for a patient, the nurse is often responsible for obtaining a sample of exudate for culture. What information will this provide?

a. Whether a patient has an infection.
b. Where an infection is located.
c. The type of cells that are being utilized by the body to attack an infection.
d. The specific type of pathogen that is causing an infection.

A

d. The specific type of pathogen that is causing an infection.

Rationale: People can transmit pathogens even if they don’t currently feel ill. Some carriers never experience the full symptoms of a pathogen. A CBC will identify that the patient has an infection.
Inspection and radiography will help identify where an infection is located. The CBC with differential will identify the white blood cells being used by the body to fight infection.
The culture will grow the microorganisms in the sample for identification of the specific type of pathogen.

148
Q

The nurse is caring for a patient with a diagnosed case of Clostridium difficile. The nurse expects to implement which of the following interventions? (Select all that apply.)

a. Administration of protease inhibitors
b. Use of personal protective equipment
c. Patient teaching on methods to inhibit transmission
d. Preventing visitors from entering the room
e. Administration of intravenous fluids
f. Strict monitoring of intake and output

A

b. Use of personal protective equipment
c. Patient teaching on methods to inhibit transmission
e. Administration of intravenous fluids
f. Strict monitoring of intake and output

Rationale: Protease inhibitors are used for treatment of viral infections, not bacterial infections. The nurse wants to protect visitors from exposure to the bacteria and protect the patient from secondary infection while immunocompromised, but the patient will need the support of family and close friends.
Contact isolation precautions must be strictly followed along with the use of personal protective equipment and teaching on methods to inhibit transmission to help break the chain of infection.
Intravenous fluids and strict intake and output monitoring will be important for the patient suffering the effects of Clostridium difficile, because it causes diarrhea with fluid loss.

149
Q

Individuals of low socioeconomic status are at an increased risk for infection because of which of the following? (Select all that apply.)

a. Uninsured or underinsured status
b. Easy access to health screenings
c. High cost of medications
d. Inadequate nutrition
e. Mostly female gender

A

a. Uninsured or underinsured status
c. High cost of medications
d. Inadequate nutrition

Rationale: Individuals of low socioeconomic status tend to be part of the underinsured or uninsured population. Lack of insurance decreases accessibility to health care in general and health screening services specifically.
High costs of medication and nutritious food also make this population at higher risk for infection. Gender has not been shown to be an increased risk factor for infection in the lower socioeconomic population.

150
Q

The nurse recognizes which term to identify the second line of defense that leads to local capillary dilation and leukocyte infiltration?

a. Normal flora
b. Inflammatory response
c. Immune response
d. Humoral immunity

A

b. Inflammatory response

Rationale: The second line of defense is the inflammatory response. Inflammation is a local response to cellular injury or infection that
includes capillary dilation and leukocyte infiltration. Normal flora is the body’s first line of defense.
The immune response is the body’s attempt to protect itself from foreign and harmful substances.
Humoral immunity is a defense system that involves white blood cells (B lymphocytes) that produce antibodies in response to antigens or pathogens circulating in the lymph and blood.

151
Q

The nurse knows that the antigen-antibody reaction is an example of what type of immunity?

a. Humoral
b. Cellular
c. Innate
d. Passive

A

a. Humoral

Rationale: Humoral immunity is a defense system that involves antibodies and white blood cells that are produced to fight antigens.
Cellular immunity involves defense by white blood cells against any microorganisms that the body does not recognize as its own.
The innate (nonspecific) immune system provides immediate defense against foreign antigens. Passive immunity occurs when a person receives an antibody produced in another body.

152
Q

The nurse uses what term to identify a disease-causing organism?

a. Pathogen
b. Normal flora
c. Germ
d. Microorganism

A

a. Pathogen

Rationale: Infectious agents include any disease-causing agent and are called pathogens. They include bacteria, fungi, viruses, and parasites.
Normal flora is a group of non–disease-causing microorganisms that live in or on the body. Germ is a term used for microorganism.
A microorganism is bacteria, fungi, or protozoa.

153
Q

The nurse is explaining to the patient why antibiotics are being administered. The answer would be correct if the nurse stated antibiotics are effective against which microorganism?

a. Viruses
b. Fungi
c. Parasites
d. Bacteria

A

d. Bacteria

Rationale: Antibiotics are effective against bacteria, and exact antibiotic sensitivity is tested so that appropriate antibiotic are prescribed.
Infections that are caused by fungi are treated with antifungal medications.
Certain antiviral medications are used to manage the symptoms of a viral infection. These medications, if given during the early phases of illness, can decrease the amount of time that the patient has viral symptoms. Treatment for parasitic infections varies depending on the type of parasite.

154
Q

The nurse anticipates correctly that what medication category would be ordered to treat athlete’s foot?

a. Antiviral
b. Antibiotic
c. Antihelminth
d. Antifungal

A

d. Antifungal

Rationale: The nurse would expect to treat athlete’s foot with an antifungal because it is a fungal infection. An antibiotic treats bacterial infections, antivirals treat viral infections and antihelminth treats parasitic worms.

155
Q

The nurse recognizes that the stethoscope most correctly represents which possible link in the chain of infection?

a. Source
b. Portal of exit
c. Portal of entry
d. Mode of transmission

A

d. Mode of transmission

Rationale: The stethoscope would be a means for the pathogen to travel from source to host. The source is the reservoir or host. The portal of exit is where the pathogen escapes from the reservoir of infection, and the portal of entry is where the microorganism enters the susceptible host.

156
Q

The nurse is teaching a group of patients about diseases that are transmitted by ticks. Which term would the nurse use when identifying the function of a tick in spreading disease?

a. Vectors
b. Bacteria
c. Viruses
d. Fungi

A

a. Vectors

Rationale: Vectors carry pathogens from one host to another. Bacteria are single-cell organisms. Viruses are the smallest organisms. Fungi are single-cell organisms that can cause infection.

157
Q

What response would the nurse provide to correctly identify the most effective method to prevent hospital-acquired infections?

a. Use of sterile technique
b. Isolation protocols
c. Antibiotic use
d. Handwashing

A

d. Handwashing

Rationale: Handwashing is the most effective method to prevent hospital-acquired infections. Sterile technique is only used for certain procedures and isolation protocols are used for patients already infected or for protective isolation in immune-compromised patients and are not used for every patient. Antibiotics are used to treat infections.

158
Q

The nurse correctly identifies which patient as having the greatest risk for infection?

a. An 80-year-old male with an enlarged prostate
b. A 24-year-old female long-distance runner
c. A 50-year-old obese male
d. A 40-year-old sexually active female

A

a. An 80-year-old male with an enlarged prostate

Rationale: The 80-year-old male has more risk factors because he is elderly and has increased risk of urinary tract infection related to prostate enlargement, so he has two risk factors.
A 24-year-old female runner is likely healthy with no additional risk factors. The 50-year-old obese male has one additional risk factor. The 40-year-old sexually active female may not have additional risk factors if she is using protection and does not have multiple partners.

159
Q

A patient admitted after abdominal surgery has a Nursing diagnosis of risk for infection. The nurse identifies which goal to be most appropriate?

a. Patient will ambulate length of hallway this shift.
b. Patient will consume 20% of meals by the end of the week.
c. Patient’s incision will be without signs or symptoms of infection at discharge.
d. Patient will verbalize need to stop antibiotics medication when symptom free.

A

c. Patient’s incision will be without signs or symptoms of infection at discharge.

Rationale: Maintaining skin integrity is an appropriate goal for this patient to ensure the patient does not develop a wound infection.
Ambulating will assist in preventing skin breakdown be getting the patient out of bed, but it is not the priority goal for a patient with an incision.
Consuming only 20% of meals will not ensure adequate nutrition and verbalizing the end of antibiotic administration to be when symptoms end is inappropriate. Antibiotics should be taken until the prescription is complete.

160
Q

The nurse recognizes which situation to be inappropriate to use alcohol-based hand sanitizer?

a. Patient with pneumonia
b. Patient with Clostridium difficile
c. Status post-appendectomy
d. Patient with HIV

A

b. Patient with Clostridium difficile

Rationale: Soap and water must be used to thoroughly clean hands if there is any visible soiling or dirt and with certain infections such as Clostridium difficile and vancomycin-resistant enterococci when preparing for a sterile or surgical procedure, before and after
eating, and after using the restroom. In the other situations, a hand sanitizer is as effective as soap and water.

161
Q

The nurse is preparing to perform suctioning on a new tracheostomy with the potential for forceful expulsion of secretions and identifies what PPE (personal protective equipment) should be worn?

a. Gloves and eyewear
b. Gloves, gown, and mask
c. Eyewear and gown
d. Eyewear, mask, gown, and gloves

A

d. Eyewear, mask, gown, and gloves

Rationale: Use gloves routinely when blood or body fluid might be present. If splashing is possible, use your nursing judgment about what other PPE might be necessary. Forceful expulsion of secretions would require all PPE—gown, mask, eyewear, and gloves—to
provide adequate protection.

162
Q

Which isolation precaution should the nurse implement for the patient who has been diagnosed with hepatitis A?

a. Airborne
b. Contact
c. Droplet
d. Protective

A

b. Contact

Rationale: Contact precautions are used when a known or suspected contagious disease may be present and is transmitted through direct contact with the patient or indirect contact with items in the patient’s environment. Airborne precautions are used when known or
suspected contagious diseases can be transmitted by means of small droplets or particles that can remain suspended in the air for prolonged periods.
Droplet precautions are used when known or suspected contagious diseases can be transmitted through large droplets suspended in the air. Protective isolation is used for patients who have compromised immune systems.

163
Q

When the patient is diagnosed with pertussis, which isolation precaution should the nurse implement?

a. Droplet
b. Airborne
c. Contact
d. Protective

A

a. Droplet

Rationale: Droplet precautions are used when known or suspected contagious diseases can be transmitted through large droplets suspended in the air.
Contact precautions are used when a known or suspected contagious disease may be present and is transmitted through direct contact with the patient or indirect contact with items in the patient’s environment. Airborne precautions are used when known or suspected contagious diseases can be transmitted by means of small droplets or particles that can remain suspended in the air for prolonged periods.

164
Q

When teaching a student nurse about removing PPE, the nurse would include which correct order of equipment removal?

a. Gloves, eyewear, gown, and mask
b. Mask, eyewear, gown, and gloves
c. Gown, mask, eyewear, and gloves
d. Gloves, gown, mask, and eyewear

A

a. Gloves, eyewear, gown, and mask

Rationale: When removing PPE, gloves, which are contaminated, are removed first to prevent contamination of the face and eyes during
removal of the mask and to prevent spread of microorganisms. Eyewear should then be removed, followed by the gown and finally the mask.

165
Q

When the nurse is wearing sterile gloves, which action would result in the gloves becoming nonsterile?

a. Fold gloved hands until procedure begins.
b. Change a dressing using aseptic technique.
c. Place sterile gloved hands below waist.
d. Use correct protocol when donning sterile gloves.

A

c. Place sterile gloved hands below waist.

Rationale: Once the hands have been placed below the waist, they can longer be considered sterile or free from organisms. Asepsis refers to freedom from disease-causing contamination. All other choices maintain asepsis.

166
Q

The nurse is planning care for an elderly patient. The nurse recognizes the patient is at risk for respiratory infections based on which factors? (Select all that apply.)

a. Decreased cough reflex
b. Decreased lung elasticity
c. Increased activity of the cilia
d. Abnormal swallowing reflex
e. Increased sputum production

A

a. Decreased cough reflex
b. Decreased lung elasticity
d. Abnormal swallowing reflex

Rationale: The elderly are at an increased risk for respiratory infections because of decreased cough reflex, decreased elastic recoil of the lungs, decreased activity of the cilia, and abnormal swallowing reflex. They do not generally have increased sputum production.

167
Q

The nurse is providing education to a patient who is being discharged home on antibiotic therapy. Which statement(s) by the patient indicates further education is needed? (Select all that apply.)

a. “I should take antibiotics every time I am sick.”
b. “I should take all antibiotics as prescribed.”
c. “I should save all unused antibiotics.”
d. “I should stop taking antibiotics when I feel better.”
e. “If I develop a rash while taking these I will call the provider.”

A

a. “I should take antibiotics every time I am sick.”
c. “I should save all unused antibiotics.”
d. “I should stop taking antibiotics when I feel better.”

Rationale: The overuse of antibiotics and inappropriate use, such as not completing prescriptions and sharing antibiotics, has led to increased resistance. Taking antibiotics as prescribed helps to ensure the infection will be treated correctly. A rash may indicate an allergic
reaction and the patient needs to report this to the provider.

168
Q

The nurse recognizes which statements by the student nurse regarding handwashing indicate a need for further education? (Select all that apply.)

a. Wash hands first, then wrists.
b. Rinse from fingertips to wrists.
c. Dry using a scrubbing motion.
d. Turn off faucet with clean, dry paper towel.
e. Dry the hands in the same order as washing them

A

a. Wash hands first, then wrists.
b. Rinse from fingertips to wrists.
c. Dry using a scrubbing motion.

Rationale: When washing hands, first wet the wrists and hands; with fingers pointing downward, first wash the wrists and then the hands below the wrists. Then apply soap, lather, and rub using a circular motion for 15 to 20 seconds.
When rinsing, rinse from wrist to fingertips, keeping hands with fingers pointing downward. Using clean paper towels, dry thoroughly in the same order (from wrists to fingers) using a patting motion. Turn off the faucet with a clean, dry paper towel.

169
Q

The nurse knows that standard precautions are indicated for which group(s) of patients? (Select all that apply.)

a. All patients
b. Patients with HIV
c. Patients with MRSA
d. Patients with tuberculosis
e. Patients who are bleeding

A

a. All patients
e. Patients who are bleeding

Rationale: The nurse can take steps at any link in the chain to halt the spread of infection.
Standard precautions are used with all patients to limit direct exposure to blood and body fluids.
The other choices are additional precautions such as airborne precautions are used with patients who have diseases such as tuberculosis and contact precautions with patients who have MRSA.

170
Q

The patient is on protective precautions. The nurse knows which statements are true regarding these precautions? (Select all that apply.)

a. A positive-pressure room with a HEPA filtration system is required.
b. Special respirator masks should be available and one size fits all.
c. No live plants are allowed in the room.
d. The patient may eat any foods desired.
e. Everyone entering the room wears a mask.

A

c. No live plants are allowed in the room.
e. Everyone entering the room wears a mask.

Rationale: Protective precautions may require a positive-pressure room. No live plants, fresh flowers, fresh raw fruit or vegetables, sushi, or blue cheese may be brought into the room because they may harbor bacteria and fungi.
The patient cannot eat just any foods because some are restricted. A mask is required for anyone entering the room and for the patient if leaving the room.

171
Q

The nurse is caring for a patient who is comatose. When performing oral hygiene, which interval is most appropriate?

a. Every shift
b. Twice daily
c. Every 4 hours
d. Daily

A

C. Every 4 hours

Rationale: Oral care should be performed every 4 hours to prevent the colonization of bacteria. Less often than every 4 hours is not effective.

172
Q

The nurse knows which skill does not require the use of sterile technique?

a. NG tube insertion
b. Foley catheterization
c. Tracheostomy care
d. PICC line insertion

A

A. NG tube insertion

Rationale: NG tube insertion requires a clean, not sterile, technique as the gastrointestinal tract is not sterile. Use strict aseptic technique when inserting an intravenous (IV) or Foley catheter and when performing suctioning of the lower airway.

173
Q

A nurse is caring for a client who has methicillin-resistant Staphylococcus aureus (MRSA) infection cultured from the urine. What action by the nurse is most appropriate?

a. Prepare to administer vancomycin.
b. Strictly limit visitors to immediate family only.
c. Wash hands only after taking off gloves after care.
d. Wear a respirator when handling urine output.

A

a. Prepare to administer vancomycin.

174
Q

A hospitalized client is placed on Contact Precautions. The client needs to have a computed tomography (CT) scan. What action by the nurse is most appropriate?

a. Ensure that the radiology department is aware of the Isolation Precautions.
b. Plan to travel with the client to ensure appropriate precautions are used.
c. No special precautions are needed when this client leaves the unit.
d. Notify the primary health care provider that the client cannot leave the room.

A

a. Ensure that the radiology department is aware of the Isolation Precautions.